BCBA Mock Exam 5 — 185 Real Exam Questions to Crush the Test (No Signup)

by

in

Getting ready for your BCBA exam? You’re in the right place.

I created RBTExamPrep.com to give you the most realistic BCBA mock exam experience possible 185 questions designed to feel just like the real thing.

My goal isn’t just to help you pass, but to help you understand every concept deeply.
Whether you get a question right or wrong, you’ll see detailed feedback explaining why, so you’ll be ready for that type next time. I want you to walk into test day feeling confident, calm, and prepared.

Many students have shared that these questions felt almost identical to the real exam and that’s exactly what I was aiming for. I’d love to hear how you did please share your score in the comments below! It really helps encourage others who are preparing for the exam. 🙂

I built this site to keep high-quality BCBA prep resources free and accessible for everyone, which is why it’s supported by ads. If it helped you, it’d mean a lot if you shared it with your peers.

If you want to keep practicing, check out the BCBA section for more tests and study materials!

💡 Tip: Like this site?
Bookmark this site using Ctrl + D or tap ‘Add to Favorites’ on your mobile browser.
 

Results

#1. A behavior analyst is teaching a complex academic skill to a client with a history of making frequent errors. To minimize the practice of incorrect responses and ensure efficient learning, the analyst decides to implement an errorless prompting procedure. Based on the principles discussed in the text, which of the following scenarios best exemplifies the effective application of errorless prompting?

The text emphasizes that errorless prompting literally means no errors We are preventing errors from occurring The goal is to preventing mistakes from happening In option C the analyst proactively intervenes before the client can commit an error on the permanent product homework By requiring a verbal preresponse and correcting it until it is accurate the client is only permitted to record the correct answer This method ensures that the client does not practice or record incorrect responses aligning perfectly with the definition of errorless prompting Options A B and D while involving prompts do not inherently guarantee error prevention For example in A fading too quickly could lead to errors in B proximity does not eliminate the possibility of an error and in D a gestural prompt might be ignored or the client might still select the wrong item before the prompt is effective allowing an error to occur

#2. Amanda and her friends are playing a drinking game. If a player flashes the wrong signal, they are immediately taken out of the game until the next round and are required to sit and quietly observe their friends continue to play. If this procedure is implemented to decrease the future occurrence of flashing the wrong signal, which specific type of time-out procedure is being utilized when a player is removed from participation and made to observe?

The scenario describes a timeout procedure where contingent upon an undesirable behavior flashing the wrong signal the individual is removed from the reinforcing activity playing the game but remains within the same environment observing others participate This specific nonexclusionary timeout procedure is known as contingent observation The individuals access to reinforcement participating in the game is temporarily removed but they are still able to observe the ongoing activity Planned ignoring involves withholding attention a potential reinforcer for a behavior but the person is not typically physically removed from an activity or situation Withdrawal of a positive reinforcer generally refers to taking away a specific item or activity that was previously available rather than removing the person from the entire context of the reinforcing environments participation While participation in the game is reinforcing the defining characteristic of this procedure is the removal from active participation with continued observation The timeout ribbon procedure involves using a tangible marker such as a ribbon to indicate when a child is in or out of timeout which is not described as part of this games rules

#3. A Board Certified Behavior Analyst (BCBA) is contracted by an adult residential home to provide behavioral consultation services for their clients. Upon observing the environment and client interactions, the BCBA identifies several practices that are inconsistent with the BACB’s Ethical Code for Behavior Analysts, potentially compromising client dignity and safety. The adult residential home is not an ABA agency and does not have behavior analysts on staff. What is the BCBA’s most appropriate first course of action in this situation?

This scenario directly tests the ethical obligations of a BCBA when confronted with unethical practices in a thirdparty setting that does not fall under the direct jurisdiction of the BACBs ethical code According to the BACBs Professional and Ethical Compliance Code behavior analysts are primarily obligated to promote the welfare of clients and to address ethical concerns proactively Option A Immediately terminate the contract While withdrawing services might be a necessary eventual outcome it is not the first or most appropriate step Immediately terminating the contract without attempting to resolve the issues first means the BCBA is not attempting to improve the situation for the clients potentially leaving them in an unchanged unethical environment The goal is to first try and correct the wrongs Option B Report to the BACB This option is incorrect because the BACBs jurisdiction is primarily over certified behavior analysts and ABA agencies An adult residential home that does not employ BACBcertified professionals or operate as an ABA agency is generally outside the BACBs direct enforcement capabilities regarding its general operational practices They are typically bound by state laws and human rights standards not the specific ethical code for behavior analysts Option D Continue services while documenting This option is unacceptable and unethical Continuing to provide services in an environment where unethical practices are occurring especially if they compromise client dignity and safety would make the BCBA complicit and could lead to direct or indirect harm to clients A behavior analyst must not provide services in an unethical environment Option C Request correction if concerns persist withdraw services This is the most ethical and professionally responsible first course of action As highlighted in the provided text the BCBA should approach the owner or supervisor of the facility clearly point out the concerns and provide them with an opportunity to respond and implement corrective measures This approach aligns with ethical principles that prioritize client welfare and attempt resolution If these attempts are unsuccessful or if the environment remains unethical and harmful then the BCBA must withdraw their consultation services to prevent perpetuating unethical practices or risking client harm This ensures that the BCBA adheres to their ethical code by attempting to rectify the situation while also protecting clients from harmful conditions

#4. A behavior analyst is setting up an Interobserver Agreement (IOA) check for a new program. To ensure objectivity in data collection, the analyst invites a colleague, who is not involved in the client’s case and is explicitly not informed about the specific intervention conditions being implemented or the expected changes in behavior, to simultaneously record data. The colleague’s role in this scenario is best described as aan

The text defines a naive observer as someone who doesnt know why youre implementing the conditions youre implementing or why things are occurring right it makes them naive The purpose of using a naive observer for IOA is to minimize observer bias ensuring that the data is not influenced by the observers knowledge of the interventions goals or expected outcomes The text explicitly states that this practice is absolutely not unethical A thirdparty contractor implies a formal contractual agreement which is not the defining characteristic here An independent variable is a manipulated variable in an experiment not a type of observer

#5. A BCBA is reviewing a student’s data on a graph for the frequency of out-of-seat behavior across four consecutive school days. The data points plotted for Day 1, Day 2, Day 3, and Day 4 consistently show a clear and substantial downward slope. Based on this visual analysis, how would the trend of the data be most accurately described?

Visual analysis of graphed data is a fundamental skill in Applied Behavior Analysis for interpreting intervention effectiveness When data points on a graph consistently show a clear and substantial downward slope from one measurement point to the next eg from Day 1 to Day 4 it indicates a decreasing trend This means the rate frequency or magnitude of the behavior is generally reducing over time An increasing trend would show an upward slope indicating the behavior is increasing A stable trend would show data points clustering around a relatively consistent level with minimal variability A variable trend would show significant fluctuations and a lack of a clear consistent direction neither consistently increasing nor decreasing

#6. Respondent conditioning involves a different type of contingency than operant conditioning. Considering the fundamental principles of each, which statement accurately describes the primary contingency involved in respondent conditioning versus operant conditioning?

The fundamental difference between respondent classical conditioning and operant conditioning lies in the type of contingency that governs the learning process Respondent Conditioning This type of learning often associated with Pavlov focuses on involuntary reflexive behaviors respondents The core mechanism involves a stimulusstimulus SS pairing where a neutral stimulus NS is repeatedly paired with an unconditioned stimulus US that naturally elicits an unconditioned response UR Through this pairing the NS becomes a conditioned stimulus CS capable of eliciting a conditioned response CR that is similar to the UR The contingency is between two stimuli Operant Conditioning Developed largely by Skinner this type of learning focuses on voluntary behaviors operants that are influenced by their consequences The core mechanism involves a responseconsequence RC pairing often described within an antecedentbehaviorconsequence ABC framework Here a behavior response is followed by a consequence eg reinforcement or punishment that affects the future probability of that behavior occurring again under similar antecedent conditions The contingency is between a behavior and its outcome

#7. A Board Certified Behavior Analyst (BCBA) is planning an intervention for a client. Upon reviewing the potential treatment options, the BCBA identifies a significant risk the effects of one intervention might linger and influence the outcome or effectiveness of another intervention if they are implemented too closely together. Considering this potential issue, which of the following experimental designs would be most prudent to avoid in this specific scenario?

The text explicitly states that if one intervention is going to impact another intervention you might want to avoid an alternating treatment design because theyre going to rapidly be following each other An alternating treatment design also known as a multielement design involves the rapid alternation of two or more distinct interventions within a single experimental phase often presented in a randomized or counterbalanced sequence While this design is highly efficient for comparing the relative effectiveness of multiple treatments its primary limitation as highlighted in the text is its susceptibility to multiple treatment interference or carryover effects This occurs when the effects of one intervention condition extend into the subsequent condition making it difficult to isolate the true impact of each individual intervention Such carryover can obscure the functional relation between the intervention and the target behavior thereby compromising internal validity In contrast a multiple baseline design Option A across settings individuals or behaviors is often preferred when carryover effects are a concern or when a withdrawal of treatment as in a reversal design Option B is impractical or unethical A changing criterion design Option D is used for behaviors that are already in the clients repertoire and can be gradually modified

#8. After his basketball team lost a crucial game by a significant margin, Coach K immediately required all team members to run 30 laps around the court. The team members intensely disliked running laps, and this consequence was directly implemented because of their loss. What type of behavior reduction procedure did Coach K utilize in this scenario?

Contingent exercise is a form of positive punishment where an individual is required to perform an effortful response in this case running laps as a consequence for engaging in an undesirable behavior losing the game The text clearly describes this running laps is what thats a form of exercise okay and now they have to run the 30 laps because what because they lost by 10 points so running the laps is contingent on losing making it what contingent exercise Positive practice involves repeatedly performing the correct form of the target behavior Negative practice involves repeatedly performing the incorrect form of the target behavior Restitutional overcorrection involves restoring the environment to a state better than it was before the misbehavior occurred None of these align with the scenario of running laps as a consequence for losing

#9. Jermaine, a 7-year-old child, has been fully potty trained for three years and has consistently demonstrated significant progress in his self-help and independent living skills, which are approaching age-typical levels. Suddenly, over a period of five consecutive nights, Jermaine begins to wet his bed. This is a novel and unexpected behavioral change given his history of successful potty training. As a Board Certified Behavior Analyst (BCBA), what is the first and most crucial action you should take when encountering such an abrupt and unexplained regression in a client’s previously mastered skill?

The correct answer is Suggest that Jermaines parents consult a medical professional The ethical guidelines for BCBAs eg BACB Professional and Ethical Compliance Code for Behavior Analysts mandate that behavior analysts rule out medical and biological causes for sudden significant and unexplained changes in behavior before proceeding with behavioral assessment or intervention In Jermaines case a child who has been consistently potty trained for three years suddenly beginning to wet the bed multiple nights in a row is a red flag for a potential underlying medical issue eg urinary tract infection diabetes sleep apnea or other physicalphysiological causes It is outside the scope of practice for a BCBA to diagnose or treat medical conditions but it is within our ethical responsibility to recognize when a medical consultation is necessary and to make that recommendation This ensures the clients health and safety are prioritized Lets consider why the other options are incorrect Interview Jermaines parents to identify recent changes in his daily routine or environment While this would be a logical step in a comprehensive functional behavioral assessment FBA it should only occur after medical causes have been ruled out Starting an FBA without considering medical factors could delay appropriate medical treatment if a health issue is present Reimplement the original potty training procedure that was successful three years ago This is inappropriate for several reasons First it assumes the problem is behavioral without ruling out medical causes Second interventions should always be based on a current and thorough behavior assessment not simply a reuse of a past procedure as the variables maintaining the behavior might have changed or a new behavior might require a different approach Simply plagiarizing past interventions is not best practice Immediately develop and implement a punishment procedure for bedwetting This is the worst and most ethically unsound option Jumping directly to a punishment procedure without any assessment especially when a medical cause is highly suspected is unethical and potentially harmful Punishment should be a last resort implemented only after thorough assessment consideration of less restrictive alternatives and a clear understanding of the function of the behavior none of which have occurred here Furthermore it completely ignores the possibility of a medical etiology for the behavior

#10. Albert, a student known for a history of disruptive classroom behavior, consistently engages in these behaviors throughout the school day. Whenever Albert disrupts the class, his teacher implements a procedure where he is immediately sent to the principal’s office. During a recent parent-teacher conference, Albert’s father explicitly mentions that Albert genuinely enjoys spending time and interacting with the school principal. Based solely on this information and the principles of applied behavior analysis, what is the most likely function maintaining Albert’s disruptive classroom behavior?

This question requires identifying the function of Alberts disruptive behavior based on the specific information provided In Applied Behavior Analysis the function of behavior refers to the reason why the behavior continues to occur over time Behaviors are typically maintained by one of four main functions 1 Social Positive Reinforcement AttentionAccess to Tangibles The behavior produces attention from others or access to desired items or activities 2 Social Negative Reinforcement Escape The behavior allows the individual to escape or avoid an aversive situation task or demand 3 Automatic Positive Reinforcement Sensory The behavior produces a sensory consequence that is inherently reinforcing to the individual without social mediation eg rocking humming selfstimulatory behaviors 4 Automatic Negative Reinforcement The behavior alleviates an internal sensation or discomfort without social mediation eg scratching an itch taking medication to relieve pain In Alberts case when he disrupts he is sent to the principals office The crucial piece of information is Alberts fathers statement that Albert really likes the school principal This strongly indicates that access to the principal and likely the principals office which is associated with the principal is a highly preferred and reinforcing consequence for Albert Therefore his disruptive behavior is being maintained by positive reinforcement in the form of access to a preferred personplace While removal from the classroom might simultaneously occur which could in other contexts suggest an escape function the explicit statement about his enjoyment of the principals company points directly to access as the primary maintaining consequence It is essential in behavior assessment to base the identified function strictly on the information given and in this scenario the preference for the principal is explicitly stated as the key factor driving the behavior

#11. A Board Certified Behavior Analyst (BCBA) has successfully implemented a novel social skills training program in a highly controlled clinic setting, demonstrating its effectiveness with a small group of clients. The BCBA now wishes to determine if these positive findings can be applied to a broader population of individuals with diverse backgrounds and in various community-based environments, such as schools and day programs. Which of the following research strategies would most directly contribute to establishing the external validity of the social skills training program?

External validity refers to the extent to which the findings of a study can be generalized to other settings populations and conditions To demonstrate that an interventions effects are robust and apply beyond the initial controlled context researchers utilize replication strategies Systematic replication involves intentionally varying one or more aspects of an earlier study eg participants settings response measures or implementers while attempting to reproduce the original findings By replicating the social skills program in different community settings with diverse clients the BCBA would be directly testing and enhancing its external validity showing that the program works under different realworld conditions A withdrawal design ABAB primarily focuses on establishing internal validity which is the extent to which a functional relation can be demonstrated between the independent variable intervention and the dependent variable behavior within a specific controlled context It does not inherently address generalizability to different conditions or populations Technological precision ensures that the intervention procedure is described with sufficient detail and clarity so that it can be consistently implemented and replicated by others While important for fidelity and the ability to replicate technological precision alone does not guarantee that the intervention will produce the same results under different conditions or with different populations which is the core of external validity Maximizing internal validity ensures that changes in the dependent variable are indeed due to the independent variable and not extraneous factors While essential for any good study it is distinct from external validity which concerns generalizability across different conditions

#12. Tony Perkins has been following a comprehensive weight training regimen that includes a specific protein powder, an energy drink, and several dietary supplements. He finds that these items are becoming quite expensive, and he wants to scientifically determine which specific components of his regimen are actually contributing to his desired results and which are not effective, so he can eliminate unnecessary expenses. To achieve this, Tony should consult with an expert to employ which type of experimental analysis?

Tonys goal is to determine what is effective and what is not within his existing regimen which functions as a treatment package A component analysis is an experimental design specifically used to identify the necessary andor sufficient components of a treatment package This involves systematically adding or withdrawing individual components from the package to observe their individual contribution to behavior change A comparative analysis would involve comparing two or more entire distinct treatment packages A parametric analysis would focus on varying a single dimension of an independent variable eg the amount of protein powder the frequency of energy drink consumption to find the optimal level A functional analysis is an assessment method used to determine the function eg attention escape access to tangibles automatic reinforcement of a problem behavior not to evaluate the effectiveness of treatment package components

#13. A behavior analyst is conducting a study to assess the impact of a differential reinforcement intervention on reducing the frequency of disruptive vocalizations among a group of sixth-grade students. The study is being carried out during the students’ regular physical education classes and lunch periods within their school environment. Considering the context and focus of this research, which type of behavioral science is primarily being conducted?

The scenario describes an example of Applied Behavior Analysis ABA Heres why Applied Behavior Analysis ABA is characterized by its focus on socially significant behavior its application to humans and its conduct in natural realworld settings like a school during PE and lunch The goal is to study the impact of interventions eg differential reinforcement to improve human behavior The text explicitly states that ABA involves our applied use of what we learn in labs to humans to determine interventions to change Lets distinguish from the other options Experimental Analysis of Behavior EAB typically involves basic research often conducted in highly controlled laboratory settings with an emphasis on discovering fundamental principles of behavior Historically EAB often involved animal subjects The text states EAB typically takes place in a lab setting with typically animals The current scenario is in a natural school setting with humans making EAB incorrect Practice Guided by Behavior Analysis refers to the direct implementation of behavioranalytic interventions and procedures in realworld settings to effect behavior change This is the actual service delivery component where an RBT or BCBA directly applies the principles The text clarifies that Practice guided by behavior analysis is our direct intervention Its what our RBTs are doing day in and day out The scenario however describes studying the impact which is more aligned with research ABA aimed at developing effective interventions rather than just routine implementation Respondent Conditioning also known as classical or Pavlovian conditioning focuses on elicited behaviors reflexes and the association of neutral stimuli with unconditioned stimuli to produce conditioned responses The study of differential reinforcement which targets operant behavior voluntary actions is not primarily a respondent conditioning investigation

#14. An RBT is learning about the foundational principles of behavior and is trying to distinguish between respondent and operant conditioning. They understand that respondent conditioning often involves involuntary physiological responses, while operant conditioning focuses on behaviors influenced by their environmental consequences. To accurately conceptualize these two types of conditioning, which of the following best describes the characteristic contingency involved in respondent conditioning versus operant conditioning?

Respondent conditioning also known as classical or Pavlovian conditioning is characterized by a StimulusResponse SR contingency In this type of conditioning a neutral stimulus NS becomes a conditioned stimulus CS after being paired with an unconditioned stimulus US eventually eliciting a conditioned response CR The behavior response is elicited by the antecedent stimulus and there is no consequence that maintains or strengthens the behavior in the same way as operant conditioning Examples include reflexes like sneezing due to pepper or salivating to the sound of a bell after conditioning Operant conditioning in contrast is characterized by the threeterm contingency or ABC contingency an antecedent A sets the occasion for a behavior B which is then followed by a consequence C that affects the future probability of that behavior Behavior is operated on the environment to produce consequences and these consequences strengthen or weaken the future likelihood of the behavior Therefore option B accurately captures the fundamental contingency structures of both types of conditioning

#15. A behavior analyst is reviewing a client’s history and notes a consistent pattern of prompt dependency from previous ABA interventions. When designing a new program to teach novel skills, the analyst wants to select a prompting procedure that actively minimizes the risk of exacerbating this prompt dependency. Which prompting procedure would be most appropriate to implement in this scenario?

Leasttomost prompting is the most appropriate procedure when a client has a history of prompt dependency because it prioritizes the use of the least intrusive prompts first This approach encourages the learner to respond independently or with minimal assistance reducing the likelihood of becoming overly reliant on prompts By starting with less intrusive prompts eg verbal gestural and only escalating to more intrusive ones eg physical if necessary the procedure systematically fades prompts helping to prevent the client from becoming dependent In contrast mosttoleast prompting starts with the most intrusive prompt which could reinforce prompt dependency Graduated guidance while quickly fading prompts is a physical prompting procedure making it highly intrusive initially Modeling is less intrusive than physical prompts but still more intrusive than some initial prompts in a leasttomost hierarchy and could still lead to dependency if not carefully faded especially for a client with a history of prompt dependency The goal is to avoid overintrusive and elongated prompts to foster independence

#16. A high school student, hoping to gain teacher approval, dedicates extra time after school for three weeks to review notes. When the teacher consistently fails to acknowledge these efforts, the student eventually stops staying late. How would this change in the student’s behavior be primarily classified in terms of its origin and the behavioral process involved?

The students behavior change is best described as ontogenic and an instance of extinction Ontogenic behavior refers to behavior that is acquired or modified during an individuals lifetime through learning experiences and interaction with their environment The students decision to stop staying late based on the teachers consistent lack of acknowledgement over three weeks is a direct result of their personal learning history not an inherited trait Phylogenic behavior refers to behaviors that are inherited as a result of the evolutionary history of a species they are genetically determined eg reflexes fixed action patterns The students study habits are not a phylogenic trait Extinction occurs when a previously reinforced behavior is no longer followed by the reinforcing consequence resulting in a decrease in the frequency of the behavior in the future In this scenario the students behavior of staying late was presumably maintained by the hope of teacher approval reinforcement When this reinforcement was withheld teacher doesnt acknowledge the student the behavior decreased student stops staying late which is the definition of extinction Respondent behavior or classical conditioning involves the elicitation of reflexes by antecedent stimuli eg salivating to a bell after pairing it with food The students voluntary action of staying late is an operant behavior maintained by its consequences not a reflex Cultural behavior refers to practices or customs learned and transmitted within a social group often verbally While social context plays a role the primary mechanism described is individual learning history and contingency management

#17. A registered behavior technician (RBT) is implementing an intervention for a child who consistently throws toys from the play area onto the floor, scattering them widely. As part of the intervention, when the child throws toys, the RBT immediately requires the child to pick up all the thrown toys, arrange them neatly back on the shelves, and then also organize and clean a different, previously tidy, section of the play area. This additional cleaning task ensures that the environment is left in a significantly better condition than it was before the toy-throwing incident. What specific type of overcorrection is the RBT employing?

Restitutional overcorrection is a form of positive punishment in which contingent on an inappropriate behavior the individual is required to restore the disrupted environment to its original state and then improve it to a condition better than it was before the behavior occurred In this scenario not only is the child required to pick up the thrown toys but also to clean and organize an additional section of the play area making the environment significantly better than it was before the misbehavior This aligns directly with the definition provided in the text regarding the high school seniors who had to remove super glue and sweep the floors making the environment better than it was before Positive practice overcorrection involves requiring the individual to repeatedly perform a correct or appropriate behavior eg repeatedly walking correctly after running in the hall Negative practice overcorrection involves repeatedly performing the inappropriate behavior itself eg repeatedly throwing paper towels in the toilet Punishment by contingent exercise requires the individual to perform an effortful response that is unrelated to the misbehavior such as doing pushups or standing upsitting down repeatedly which is not what is described here as the task is directly related to cleaning the environment

#18. A Board Certified Behavior Analyst (BCBA) is implementing an A-B-A-B reversal design to evaluate the effectiveness of a new social skills intervention for a client. After an initial baseline phase (A), the intervention is introduced (B), leading to a significant increase in the client’s social interactions. The BCBA then systematically withdraws the intervention, returning to baseline conditions (second A phase), and observes a subsequent decrease in social interactions, which return to levels similar to the initial baseline. This specific withdrawal of the intervention and the observed change in behavior primarily serves to fulfill which component of baseline logic?

The ABAB reversal design is a powerful experimental design used to demonstrate functional control It involves alternating phases of baseline A and intervention B The critical component for demonstrating functional control specifically addressed by withdrawing the intervention and observing a return to baseline levels is verification Verification occurs when after the independent variable intervention has been shown to affect the dependent variable behavior the independent variable is withdrawn and the behavior reverts to its original baseline levels This observed return to baseline significantly increases the probability that the observed changes during the intervention phase were indeed due to the independent variable and not due to extraneous variables If the behavior does not return to baseline it would suggest that other factors might be at play or that the intervention produced a lasting effect making verification of functional control more complex within this specific design Prediction involves observing stable baseline data to predict that the behavior would remain at that level if no intervention were applied Replication involves reintroducing the intervention and observing a return to the desired behavior change further strengthening the evidence of functional control Stability refers to a consistent level of responding in a given phase which is a prerequisite for making accurate predictions and demonstrating control but not a component of baseline logic itself in the same way as prediction verification and replication

#19. Joshua is a member of a research team at a local university. For five months, he and his team have been meticulously studying the impact of controlled deprivation on the behavioral patterns of mice housed in a shared laboratory environment. Following their investigation, Joshua and his team compiled and published a detailed report of their experimental findings. Based on this description, in which specific branch of behavior analysis is Joshua participating?

The text clearly outlines the distinctions between the four branches of behavior analysis Joshuas activities involving a research team laboratory setting animal subjects mice and the study of fundamental behavioral principles impact of deprivation are all hallmark characteristics of the Experimental Analysis of Behavior EAB EAB focuses on basic research often in controlled environments to understand the fundamental principles of behavior Applied Behavior Analysis ABA typically involves research and intervention in natural environments addressing socially significant human behaviors Conceptual Analysis of Behavior involves the theoretical and philosophical discussions about behavior such as reviewing and interpreting behavior analytic principles Practice Guided by Behavior Analysis refers to the direct application and implementation of behavior analytic principles in realworld settings with clients often by practitioners or technicians Since Joshua is conducting controlled research with animals in a lab EAB is the most accurate classification

#20. A research assistant is designing an intervention for a client’s maladaptive behavior. After meticulously collecting baseline data, she hypothesizes that her proposed intervention will reduce the frequency of the target behavior. She predicts the specific changes she expects to see once the intervention is implemented next week and aims to eventually demonstrate experimental control between her intervention (IV) and the client’s behavior (DV). In the context of the levels of scientific understanding in Applied Behavior Analysis, the research assistant’s current activities of hypothesizing and anticipating outcomes primarily exemplify which level?

The text explicitly states that the research assistant is hypothesizing possible correlation and predicting what may happen once she starts introducing her intervention Prediction as a level of scientific understanding involves establishing that two events regularly covary allowing one to predict the occurrence of one event based on the presence of another This stage involves formulating hypotheses about potential relationships and anticipating future outcomes While the ultimate goal in ABA is to achieve control by demonstrating a functional relation between an independent variable and a dependent variable the act of predicting outcomes before the intervention is fully implemented and control is established falls squarely within the prediction stage

#21. A teacher observes that a student, Jackie, who previously engaged in frequent attention-seeking behaviors, has significantly improved her on-task behavior and is now consistently following classroom rules. As a result, the teacher begins to allocate more attention to other students who require assistance and, unintentionally, less to Jackie. Over time, Jackie’s newly acquired appropriate behaviors start to decrease in frequency. Based on this scenario, which of the following ABA principles is most likely occurring with Jackie’s appropriate behavior?

This scenario most accurately describes unintentional extinction Extinction occurs when reinforcement for a previously reinforced behavior is withheld leading to a decrease in the future frequency of that behavior In this case Jackies appropriate behaviors were likely maintained by teacher attention When the teacher unintentionally started paying less attention to Jackie after her behavior improved she was withdrawing reinforcement for the appropriate behavior thus putting it on extinction This often happens inadvertently when a learners behavior improves and the focus shifts but its crucial to continue reinforcing desired behaviors to maintain them Planned ignoring is a specific extinction procedure that is intentionally implemented to decrease a problem behavior by withholding attention or other identified reinforcers The key here is that its planned and typically targets undesired behaviors In this scenario the ignoring is unintentional and is affecting desired behaviors Punishment involves adding an aversive stimulus or removing a desirable one to decrease a behavior There is no indication of an aversive stimulus being added or a desirable stimulus being removed contingently to decrease Jackies behavior rather reinforcement attention is simply being withheld which is characteristic of extinction Continuous reinforcement would mean the teacher is providing reinforcement attention for every instance of Jackies appropriate behavior which is explicitly stated not to be occurring

#22. At McNeil Law Firm, account executives historically received a monthly bonus for completing five or more case briefs. This month, an announcement was made that the requirement for the bonus would now be 15 or more case briefs. Following this change, the firm recorded the lowest number of bonuses ever given in a single month. From an ABA perspective, what is the most likely reason for the observed decrease in bonuses?

The most likely explanation for the significant decrease in bonuses following a dramatic increase in the response requirement for earning them is ratio strain Ratio strain occurs when the response requirement the ratio of responses to reinforcement is increased too quickly or becomes too large leading to a decrease or cessation of responding In this scenario the requirement jumped from 5 to 15 case briefs which is a substantial and abrupt increase in the effort required to earn the bonus This often leads to a breakdown in behavior as the effort no longer seems worth the reinforcement or the behavior simply extinguishes due to insufficient reinforcement for the increased effort Many of the account executives quit is not supported by the provided text and introduces outside information We should stick to what is stated in the scenario The bonus measurement system was implemented incorrectly is also not indicated in the text Theres no information suggesting an issue with how bonuses are measured only with the contingency for earning them The announcement of the new requirement was misunderstood is not a behavioranalytic explanation While misunderstanding can occur ABA focuses on observable behavior and environmental contingencies The core issue described is a change in the reinforcement schedule and its effect on response rate which points directly to ratio strain

#23. As a BCBA supervising Registered Behavior Technicians (RBTs), you are responsible for ensuring the competence of your supervisees. When training a new RBT on a specific discrete trial teaching procedure, what is your primary ethical and professional obligation before allowing the RBT to implement this procedure independently with a client?

The text explicitly states our responsibility to the clients to be sure our training is adequate Remember youre responsible for your rb Ts for your supervisees So if youre training on something make sure its right and Known before you allow them to do it This highlights that its not enough to simply provide training materials A or observe the supervisor C or even just verbally confirm understanding D The supervisors primary obligation is to ensure the RBTs training is adequate and that they have demonstrated the skill correctly and proficiently implying observed practice and competence before independent implementation This ensures client safety and effective service delivery

#24. Glenn is working with a new client who consistently demonstrates high motivation for Hershey Kisses. Glenn proposes using chocolate as a reinforcer to facilitate skill acquisition. However, the client’s parents express significant concern about the sugar content and potential for cavities, explicitly stating they do not want sugary treats used as reinforcers. How should Glenn, as a Board Certified Behavior Analyst (BCBA), ethically and effectively proceed in this situation, considering the welfare of the client and the concerns of the stakeholders?

This scenario presents a common ethical dilemma encountered by BCBAs regarding stakeholder preferences and the selection of reinforcers According to the BACB Professional and Ethical Compliance Code for Behavior Analysts practitioners must prioritize client welfare 20 Client Welfare act with responsible conduct 201 Responsible Conduct of Behavior Analysts and collaborate effectively with stakeholders 209 BehaviorAnalysts Ethical Responsibilities to the Client Parents as primary stakeholders have a right to express concerns about their childs health and to be involved in treatment planning Option B Correct This is the most ethical and procedurally sound approach By proposing temporary use of chocolate while simultaneously pairing it with praise and tokens Glenn is actively working to establish conditioned secondary reinforcers This strategy allows for initial skill acquisition using a known effective reinforcer while proactively fading out the less desirable primary reinforcer and addressing the parents health concerns in the long term This approach demonstrates professional flexibility adherence to ethical guidelines by respecting parental concerns and a commitment to building a sustainable reinforcement system Option A Incorrect Proceeding with an intervention despite explicit parental protest especially concerning health risks is a direct violation of ethical principles related to stakeholder collaboration and respecting the rights of the client via their guardians It undermines the therapeutic relationship and can lead to a breakdown in trust Option C Incorrect Terminating services due to a reasonable parental objection eg concerns about health is unprofessional and unethical It reflects an inability to adapt treatment plans collaborate and problemsolve potentially abandoning the client without exploring alternative solutions or making appropriate referrals Option D Incorrect While finding alternative reinforcers is necessary simply assuming toys will be reinforcing without conducting a proper preference assessment or reinforcer assessment is not datadriven practice The text only states chocolate is a reinforcer not toys This approach is speculative and does not guarantee treatment effectiveness or respect the principles of individualized assessment

#25. As a BCBA, you are committed to upholding the BACB Ethical Code of Conduct, which includes avoiding dual relationships with clients and their families. However, you also recognize the importance of establishing an environment conducive to effective services. You regularly engage with clients and stakeholders by communicating openly, respecting cultural differences, and participating in polite conversation. What is the primary purpose of these professional behaviors in the context of service delivery?

The text explicitly states that communicating with clients respecting their culture and engaging stakeholders in polite conversation are examples of building rapport Rapport is a close and harmonious relationship in which the people or groups concerned understand each others feelings or ideas and communicate well While BCBAs must avoid dual relationships and maintain professional boundaries developing a positive and trusting relationship with clients and their families stakeholders is essential for effective service delivery This trust and cooperation built through rapport are foundational for successful assessment intervention implementation and generalization of skills Without rapport stakeholders may be less inclined to follow through with recommendations and clients may be less motivated to participate Option A is incorrect as rapport building does not directly address dependency Option B is incorrect because rapport facilitates rather than replaces protocol adherence Option D while some data might be informally gathered it is not the primary purpose of these relationshipbuilding actions the primary purpose is the positive relationship itself which then supports all other aspects of intervention

#26. Consider the stimuli birds and kites. Although they possess no common physical features or topographical similarities, they both reliably evoke the response things that fly in a particular individual. Based on the principles of Applied Behavior Analysis, what type of stimulus class do birds and kites represent in this scenario?

A stimulus class is a group of stimuli that have a common effect on behavior The text explicitly defines an arbitrary stimulus class as one where stimuli do not share common topographical similarities but still evoke the same response In contrast a feature stimulus class also known as a conceptual stimulus class consists of stimuli that share common physical features eg color shape size A response class refers to a group of responses that produce the same effect on the environment regardless of their topographical differences eg opening a door by pushing pulling or turning a knob A consequence class refers to stimuli that follow a behavior and affect its future probability not antecedent stimuli that evoke a response Since birds and kites lack shared physical features but evoke the same response they belong to an arbitrary stimulus class

#27. When a behavior analyst implements a behavior-change program, they systematically add, remove, or modify components of the intervention to determine their effect on the client’s behavior. In the context of experimental design, what specific type of variable is being manipulated in this process?

The text clearly states When youre adding and removing interventions in order to control behavior youre adding and removing what independent variables The independent variable IV is the variable that the experimenter manipulates or changes In ABA interventions treatments or specific components of a program are considered independent variables because the behavior analyst systematically applies or withdraws them to observe their effect on the target behavior the dependent variable This manipulation is crucial for establishing experimental control and determining functional relations

#28. A client frequently elopes from their mother when in public spaces, such as grocery stores. To better understand this challenging behavior, a BCBA meets with the client’s mother. During this meeting, the BCBA asks the mother detailed questions about what typically happens immediately before the elopement (antecedents), what the elopement behavior looks like, and what usually occurs immediately after the behavior (consequences), including how she responds. Given this scenario, what type of data collection or assessment method is the BCBA primarily employing?

The scenario describes the BCBA gathering information about the clients behavior by interviewing a stakeholder the clients mother This method where information is obtained through verbal reports questionnaires or rating scales rather than direct realtime observation of the behavior is known as indirect assessment While the questions are focused on gathering information about antecedents behaviors and consequences ABC the act of asking a stakeholder about these events rather than directly observing them as they happen classifies it as indirect Direct ABC data collection A would involve the BCBA or another trained observer physically present and recording antecedents behaviors and consequences as they occur in the natural environment Functional analysis B is a direct experimental assessment that systematically manipulates antecedents and consequences to identify the function of a behavior Scatterplot analysis D is a form of direct data collection used to identify patterns of behavior across time eg time of day day of week but also requires direct observation or existing records

#29. A Registered Behavior Technician (RBT) is observing a young learner who is able to read aloud from a book. The learner can accurately pronounce all the words, demonstrating strong decoding skills. However, when asked specific questions about the story’s plot or characters, they cannot explain what they just read. Based on the principles of verbal behavior, specifically regarding textual operants, which of the following is NOT considered a critical component of a textual verbal operant?

The provided text explicitly states Does the learner even have to understand what they are reading Well no It further clarifies that That comprehension piece comes a little later But in order for it to be a textual the learner does not have to understand what they are reading C is not a critical component of a textual verbal operant Pointtopoint correspondence is identified as a critical component meaning the response matches the stimulus reading exactly what is seen A textual operant is defined by a visual verbal stimulus controlling a vocal response which is also a core component

#30. You teach your younger brother the ‘walk the dog’ trick with his new yo-yo. A week later, you observe him independently performing ‘around the world’ and ‘cat’s cradle’ with the same yo-yo, even though you never explicitly taught him these additional tricks. What behavioral phenomenon is being demonstrated by your brother?

The scenario described is a classic example of response generalization The text clearly defines response generalization as occurring when a single stimulus the yoyo leads to multiple untaught responses around the world and cats cradle after only one response walk the dog was directly taught This is distinct from stimulus generalization which involves one response occurring across multiple stimuli The text also clarifies that this is not behavior maintenance as maintenance refers to the continued performance of a taught behavior not the spontaneous emergence of new related behaviors

#31. A two-year-old child’s spontaneous mands for a preferred toy have decreased significantly over the past week. The parents express concern that they may have inadvertently punished the child’s communication. However, a review of their interactions reveals no direct application of aversive stimuli or removal of positive reinforcement immediately following the mands. Based on the principles of Applied Behavior Analysis and the information provided, what is the MOST likely behavioral phenomenon occurring that explains the decrease in the child’s mands?

The provided text explicitly clarifies that when theres no indication of punishment or extinction occurring a decrease in a target behavior often means that a different functionally equivalent behavior is being reinforced instead This alternative behavior then competes with the target behavior leading to its decrease While satiation could decrease motivation and extinction involves withholding reinforcement the scenario specifically negates punishment and points towards an interactional cause for the decrease making the reinforcement of an alternative behavior the most probable explanation provided within the context Regression is a developmental concern not a primary behavioral principle explanation for a recent behavioral change in this context

#32. A group of four friends agrees that if all four of them successfully hit their golf balls into the middle of the fairway on the first tee, they will collectively earn a privilege (e.g., not having to take a penalty shot). If even one person fails to hit the fairway, the entire group loses the privilege. This arrangement is an example of which type of group contingency?

This scenario describes an interdependent group contingency In an interdependent group contingency reinforcement for each member of the group is contingent upon every member of the group meeting the specified behavioral criterion The group as a whole must perform the target behavior successfully for any individual within the group to receive the reinforcement If any single member fails to meet the criterion the entire group or at least those who participated in the contingency does not receive the reinforcement This fosters peer support and shared responsibility In contrast a dependent group contingency often called a hero procedure is when the reinforcement for the entire group is contingent upon the behavior of one individual or a small subset of the group An independent group contingency is when reinforcement for each individual is contingent only on that individuals own behavior even if they are part of a group where others are also working towards the same goal

#33. Slash, a guitarist in a band, was performing a solo during a live show when he accidentally played an incorrect chord. To address this error and reduce its future occurrence, his band leader instructed him during the next practice session to intentionally play that exact incorrect chord 15 times in a row, immediately after performing the solo correctly. Which overcorrection procedure is the band leader utilizing to decrease Slash’s error?

Negative practice overcorrection is a form of overcorrection where contingent on an inappropriate behavior the individual is required to repeatedly perform the incorrect or unwanted behavior The goal is to make the inappropriate behavior aversive or tedious through repeated execution thereby decreasing its future occurrence In this case Slash is made to repeatedly play the wrong chord Positive practice overcorrection would involve repeatedly performing the correct behavior often in an exaggerated manner to build proficiency Restitutional overcorrection requires the individual to restore the environment to a state that is better than it was before the misbehavior occurred Response cost involves the removal of a specified amount of a reinforcer contingent on an inappropriate behavior which is not described here

#34. Grace, a BCBA, has occasionally felt uneasy about the safety of a client’s home environment but lacks concrete proof to support her concerns. One day, she is contacted by a local law enforcement agency that is conducting an investigation and requests specific details about her client. Grace is committed to upholding client confidentiality but also recognizes the potential seriousness of a law enforcement inquiry. According to the Professional and Ethical Compliance Code for Behavior Analysts, what is the most appropriate and ethical action for Grace to take in this situation?

The BACBs Professional and Ethical Compliance Code specifically Section 206 Maintaining Confidentiality and 207 Disclosures permits behavior analysts to disclose confidential information without client consent when mandated by law court order or when necessary to protect the client or others from harm Disclosure to law enforcement during an official investigation often falls under a legal mandate or a permissible exception particularly when the information is limited to what is relevant to the investigation Refusing to provide information without consent is generally the correct approach for other third parties but law enforcement typically has a different legal standing Reporting herself for uneasy feelings is unwarranted as theres no indication she violated a code Providing only nonidentifying information might not fulfill a legal obligation or adequately assist a legitimate investigation where specific details are requested

#35. Henry, a client, has consistently put his left shoe on first and then his right shoe on second every morning for the past four weeks. During this same four-week period, he has received two unexpected promotions at his job. Henry now firmly believes that his specific shoe-wearing routine is directly responsible for his career success. As a behavior analyst, why would you most strongly doubt that a functional relation exists between Henry’s shoe-wearing behavior and his work promotions?

This scenario describes superstitious behavior which occurs when a behavior is accidentally reinforced by an unrelated consequence leading the individual to believe there is a causal link A core principle for establishing a functional relation in behavior analysis is temporal contiguity the closer in time two events behavior and consequence occur the more likely they are to be functionally related In this case many hours or even days pass between Henrys shoewearing and his promotions making a direct functional relation highly improbable While its highly unlikely a functional relation exists stating its impossible option A is too strong as behavior analysis emphasizes empirical verification and avoiding definitive claims without data Option C describes an experimental manipulation that might be used but the initial doubt stems from the lack of temporal contiguity not the necessity of an experiment Option D is incorrect four weeks can be sufficient to observe trends but the type of relation temporal is the primary concern here

#36. Cindy, a teacher, wants her class to learn teamwork and plans to implement a group contingency for homework assignments. She is aware that her students, despite being intelligent, have recently engaged in instances of bullying other students. Considering the potential for social backlash and ethical concerns, which type of group contingency should Cindy most likely avoid?

The text explicitly states that a Dependent Group Contingency also known as a Hero Contingency is a significant ethical concern from the standpoint of potential social backlash especially if theres an existing issue with bullying In this type of contingency the entire groups reinforcement is dependent on the behavior of one or a few hero individuals If these individuals fail to meet the criteria the entire group loses reinforcement which can lead to negative social interactions including bullying directed at the hero who failed Given Cindys awareness of bullying instances avoiding this particular group contingency is crucial to prevent further social harm An Independent Group Contingency reinforces individuals based on their own performance an Interdependent Group Contingency requires all members to meet the criteria for the entire group to be reinforced and a Natural Group Contingency relies on naturally occurring reinforcement none of which carry the specific risk of singling out an individual for group failure in the same way as a Dependent Hero Contingency

#37. Kendall is teaching a new client about animals. During a session, Kendall presents a picture of a dog and asks the client to match it to an identical picture of a dog from a set of foils. The client consistently matches the dog picture to itself. According to the principles of stimulus equivalence, what type of relation has Kendall primarily demonstrated in this specific instance?

Reflexivity also known as identity matching occurs when a stimulus is matched to itself A A In this scenario the client is presented with a picture of a dog stimulus A and is required to match it to another identical picture of a dog also stimulus A The client consistently matching the dog picture to itself directly exemplifies a reflexive relation Symmetry AB BA involves the reversibility of a trained conditional relation meaning if A evokes B then B will evoke A without direct training Transitivity AB BC therefore AC involves an untrained conditional relation that is derived from two other trained conditional relations An equivalence class is a set of stimuli that are functionally equivalent but the specific demonstration of matching a stimulus to itself is the foundational relation of reflexivity

#38. In the context of single-subject experimental designs and the elements of baseline logic, which specific aspect aims to confirm what the data path would likely have looked like if the independent variable (intervention) had never been introduced, thereby strengthening the confidence in the initial prediction made about the baseline’s trajectory?

Baseline logic is a cornerstone of singlesubject experimental designs allowing researchers to demonstrate experimental control It comprises three main components prediction verification and replication Prediction is the statement of the anticipated outcome of a presently unknown or future measurement Based on stable baseline data the researcher predicts that the behavior would continue in the same pattern if no intervention were introduced Verification is the process of demonstrating that the prior level of baseline responding would have remained unchanged had the independent variable not been introduced It confirms the accuracy of the initial prediction made during the baseline phase For example in a multiplebaseline design when an intervention is applied to the first participantsettingbehavior while the baseline continues for a second the continued stable baseline for the second element verifies the prediction that the behavior would not change without the intervention Replication is the repeating of the independent variables effect on the dependent variable This demonstrates the reliability and generality of the experimental control and is often achieved by reintroducing the intervention or applying it across different subjects settings or behaviors Validation is not a core component of baseline logic It refers to ensuring that a measurement system or intervention accurately assesses or addresses what it is intended to

#39. Jeremiah, a behavior analyst, has received a new set of 40 novel sensory items he intends to use as potential reinforcers for a client. To ensure the most effective and individualized intervention, Jeremiah wants to establish a precise hierarchy of the client’s preferences for all 40 items, identifying which item is most preferred, second most preferred, and so on, down to the least preferred. Considering the objective of establishing a detailed preference hierarchy among a large number of items, which preference assessment method would be most useful and robust for Jeremiah to employ?

When the goal is to establish a comprehensive and precise hierarchy of preferences among a large number of items the Paired Choice or Forced Choice Preference Assessment is generally considered the most robust and useful method In this assessment every stimulus is presented systematically with every other stimulus in pairs The learner chooses one item from each pair and the item chosen most frequently across all pairings is identified as the most preferred followed by the next most frequently chosen and so on This method provides a clear rankordered hierarchy because it involves direct headtohead comparisons of all items While other methods like Multiple Stimulus Without Replacement MSWO can provide a rank order of selected items Paired Choice is particularly effective for establishing a highly refined hierarchy by ensuring each item is compared against every other potential reinforcer leading to a more reliable preference ranking Single Stimulus assesses engagement with individual items MSW returns items after selection making it less ideal for a full hierarchy and Free Operant observes unconstrained access to items but doesnt force a comparative choice necessary for a strict ranking

#40. A BCBA instructs three RBTs to collect duration data on a client’s ‘screaming behavior.’ The operational definition provided is ‘Any loud, piercing vocalization exceeding the typical conversational volume of peers, lasting for more than 2 seconds, and not serving a communicative function.’ RBT A reports the client screamed for an average of 10 minutes per session. RBT B reports the client engaged in ‘loud vocalizations’ for an average of 12 minutes per session. RBT C sends data indicating the client engaged in ‘property destruction’ for an average of 5 minutes per session. Based solely on this information, which quality of good data is most clearly lacking in RBT C’s report?

The text highlights three crucial qualities of good data accuracy validity and reliability Accuracy refers to measuring the true value of what occurred We cannot definitively assess accuracy here without a true value to compare RBT Cs data against Reliability refers to the consistency of measurement when repeated We typically assess reliability through interobserver agreement IOA or by comparing repeated measures from the same observer which is not provided for RBT Cs data in isolation nor can it be assessed across different behaviors Validity refers to measuring what we are actually attempting to measure The BCBAs target behavior was screaming behavior RBT C however reported data on property destruction Because RBT C measured a different behavior than what was intended their data clearly lacks validity They are not measuring what they were asked to measure Treatment fidelity while important refers to the extent to which the intervention plan is implemented as intended which is a different aspect from the quality of the behavioral data itself

#41. Jenny, a parent, wants to know how often her son is taken to the bathroom at his daycare each day. She needs a measure that can be easily compared across different days, regardless of minor variations in the total time spent at daycare. To accurately capture this information and ensure meaningful comparison, which dimensional quantity would be most useful for Jenny to track?

The question asks for how ofteneach day indicating a count of behavior occurrences within a specified period of time This is the definition of rate Rate is a measure of frequency a simple count combined with a unit of time eg per day per hour While frequency a count of occurrences is a fundamental measure it does not account for the length of the observation period making direct comparisons across sessions of varying lengths potentially misleading For example knowing a child went to the bathroom six times is less informative than knowing they went six times per day Temporal locus refers to the point in time at which behavior occurs eg latency interresponse time Duration measures the total time a behavior lasts Therefore rate is the most appropriate and useful dimensional quantity for Jenny to track as it standardizes the count by incorporating a time component allowing for accurate and comparable data across different days

#42. A behavior analyst is conducting a research study. Which term most accurately describes the particular type and sequence of conditions implemented in the study to allow for a meaningful comparison of the effects of the presence and absence of independent variables on behavior?

This question directly asks for the definition provided in the text regarding the arrangement of conditions in a study to establish a functional relation Option A Functional Analysis A functional analysis FA is a specific type of experimental design used within a functional behavior assessment FBA to directly test the hypothesized functions of challenging behavior by systematically manipulating environmental variables antecedents and consequences in analogue conditions While it involves comparing the presence and absence of independent variables experimental design is the broader more encompassing term that describes the overarching methodology for establishing causeandeffect relationships Option B Experimental Design This is the correct answer The text defines experimental design as the particular type and sequence of conditions in a study so that a meaningful comparison of the effects of the presence and absence of independent variables can be made This refers to the systematic arrangement of independent variable manipulations eg intervention present intervention absentbaseline different levels of intervention to determine a causal or functional relationship with the dependent variable behavior Examples of experimental designs in ABA include withdrawal designs multiple baseline designs alternating treatments designs and changing criterion designs Option C Functional Behavior Assessment A Functional Behavior Assessment FBA is a comprehensive process used to identify the purpose or function of a behavior It can involve indirect assessments interviews rating scales direct descriptive assessments ABC data scatterplots and functional analyses While an FBA can inform an experimental design it is a process for understanding behavior not the design itself for demonstrating experimental control over interventions Option D Narrative Recording Narrative recording eg ABC recording is a direct observation method used to describe behaviors and their environmental contexts in a descriptive prose format It is a data collection technique not an experimental design for systematically comparing the effects of independent variables across conditions

#43. When an applied behavior analyst systematically adds and removes interventions or treatments during a study, what is the primary variable they are attempting to control or influence, and whose changes they are observing?

In experimental design interventions or treatments are considered the independent variables IVs because they are systematically manipulated by the experimenter The behavior that is being measured and is expected to change as a direct result of these manipulations is the dependent variable DV The primary goal when adding and removing IVs is to establish experimental or functional control which means demonstrating that the observed changes in the dependent variable are indeed caused by the manipulations of the independent variable rather than by other factors While confounds and extraneous variables are important to identify and control they are threats to experimental control that we aim to minimize the direct target of control and observation through IV manipulation is the DV

#44. A high school cheerleader’s BCBA coach implements a self-management program to help her learn a complex routine for tryouts. The program is designed to achieve flawless performance of the entire routine, as this is necessary for her to make the team. After diligently following the intervention, the cheerleader is able to perform 80 of the routine correctly. However, this level of performance is insufficient for her to be selected for the team, meaning she did not meet her ultimate goal. Considering the seven dimensions of Applied Behavior Analysis (Baer, Wolf, Risley, 1968), which dimension does this intervention most clearly lack?

The correct answer is Effective The dimension of Effective in ABA requires that an intervention produce significant practical and socially meaningful changes in behavior In this scenario while the cheerleader showed some improvement by learning 80 of the routine this change was not significant enough to achieve the stated goal of performing flawlessly and making the team For an intervention to be truly effective the behavior change must be of practical importance to the individual and those around them and reach a level that meets the socially valid objective Learning 80 of the routine though an improvement fails to meet the criteria of social validity and significance for this individuals goal Lets consider why the other options are incorrect Behavioral The intervention focuses on an observable and measurable behavior performing a cheer routine and the coach can change the intervention to modify this behavior This dimension is present Conceptually Systematic The coach is a BCBA and the use of a selfmanagement program suggests that the intervention is derived from basic principles of behavior analysis even tracking the behavior Based on the information given it appears to be conceptually systematic Technological This dimension requires that all procedures be described clearly and concisely so that they can be replicated by anyone with the necessary training and resources The text does not provide enough information to definitively state whether the intervention lacks this dimension it only focuses on the outcome Therefore we cannot conclude it lacks this aspect

#45. New research from a reputable university suggests that a specific dietary intervention can significantly improve certain cognitive deficits associated with intellectual disabilities. In response, a team of researchers at another university decides to design and conduct their own study to independently verify these findings, using similar methodologies. They want to ensure the initial findings are robust and generalize across different populations. By undertaking this independent replication and verification of existing research, these scientists are demonstrating adherence to which fundamental assumption of science in Applied Behavior Analysis?

Philosophical doubt is a fundamental attitude of science that requires scientists to continuously question the truthfulness of all scientific knowledge It involves being skeptical and critically evaluating all findings even wellestablished ones and maintaining an open mind about the possibility of new evidence The researchers decision to retest the existing findings demonstrates this commitment to questioning and verifying knowledge Determinism is the assumption that the universe is a lawful and orderly place and that all phenomena occur as a result of other events Technological refers to the ability to precisely describe behaviorchange procedures so that others can replicate them Generality refers to the extent to which a behavior change is durable occurs in a variety of environments andor spreads to a variety of behaviors

#46. Carol, who works at a marketing firm, is tasked with selecting the newest promotional material for her clients. To gather unbiased feedback, she asks her friend Dana for assistance. Carol places four different promotional material designs on a table in front of Dana. Dana is instructed to choose the design she likes best. Once Dana makes a choice, Carol removes the three unchosen designs and immediately replaces them with three entirely new, different designs, then asks Dana to choose again from the new set of four. This process continues through multiple trials. What specific type of preference assessment strategy is Carol utilizing?

This question describes a specific method of preference assessment used in Applied Behavior Analysis The key details are the number of stimuli presented simultaneously and what happens to the stimuli after a choice is made Option D Correct A Multiple Stimulus With Replacement MSWR preference assessment involves presenting a set of three or more stimuli in this case four designs to an individual When the individual chooses an item that chosen item is typically returned to the array and the unchosen items are replaced with new items or in some variations the unchosen items are replaced while the chosen item is retained creating a new array for the next trial with a constant number of items The goal is to efficiently sample a wide range of potential reinforcers Carols procedure of removing the unchosen items and replacing them with new items while maintaining the array size perfectly aligns with the core principle of with replacement as it involves continuously introducing new stimuli into the assessment pool Option A Incorrect Paired Stimulus also known as Forced Choice preference assessment involves presenting only two stimuli at a time and recording which one is chosen Carol is presenting four designs simultaneously Option B Incorrect Multiple Stimulus Without Replacement MSWO preference assessment involves presenting a set of three or more stimuli When an item is chosen it is removed from the array for subsequent trials and the array size decreases Carol however is replacing the unchosen items with new ones thus maintaining the array size and not without replacement Option C Incorrect Single Stimulus also known as Successive Choice preference assessment involves presenting only one stimulus at a time and recording whether the individual approaches or interacts with it Carol is presenting multiple items simultaneously not one at a time

#47. Glinda, a Registered Behavior Technician (RBT), is working with a client who frequently engages in self-injurious behavior (SIB) by hitting himself in the chest with a closed fist. Her supervisor instructs her to intervene immediately to prevent injury. Glinda quickly responds by wrapping her arms around the client, physically preventing him from raising his arm and making contact with his chest. This intervention successfully stops the self-injurious behavior from occurring in that specific moment. This particular type of intervention is most accurately identified as

Response Blocking is an antecedentbased intervention strategy where the individual is physically prevented from engaging in the problem behavior In this scenario Glindas action of wrapping her arms around the client directly prevents the physical act of hitting himself It is crucial to distinguish this from Extinction Extinction involves withholding reinforcement for a previously reinforced behavior which means the behavior must occur for its reinforcing consequences to be withheld Since Glinda physically prevented the hitting behavior from occurring it was not possible to withhold reinforcement after the behavior thus it cannot be classified as extinction Punishment involves a consequence delivered after the behavior to decrease its future probability which is not what occurred here as the behavior was prevented DRI involves reinforcing a behavior that cannot occur simultaneously with the problem behavior which is a different procedural category focused on reinforcement

#48. A Board Certified Behavior Analyst (BCBA) is working with a new learner to teach them to identify various common objects. The BCBA’s primary goal is to prevent the learner from making any incorrect responses during the initial acquisition phase to avoid the development of error patterns. Which of the following prompting strategies would be the most effective for achieving errorless learning?

Errorless learning is a teaching strategy designed to prevent learners from making errors during instruction The most effective way to achieve errorless learning is by using a mosttoleast prompting hierarchy This involves starting with the most intrusive prompt necessary eg a full physical prompt to ensure the learner emits the correct response and then gradually fading the prompt intensity across trials as the learner gains proficiency The goal is to maximize correct responses from the outset and minimize opportunities for errors The text explicitly states The best answer here to ensure errorless learning is using most to least prompting Meaning were going to use the most prompting necessary to excuse me to avoid errors Options A B and D either allow for errors or do not guarantee error prevention at the initial stages of learning

#49. The text describes two primary types of conditioning that explain how stimuli affect behavior respondent and operant. Given the distinct mechanisms through which each type of conditioning occurs, which of the following accurately represents the basic contingency structure for respondent conditioning and operant conditioning, respectively?

Respondent conditioning also known as classical or Pavlovian conditioning involves a StimulusResponse SR contingency Here an antecedent stimulus directly evokes a reflexive involuntary response and there is no consequence that maintains the behavior For example sniffing pepper Stimulus directly leads to sneezing Response Operant conditioning conversely involves a StimulusResponseStimulus SRS or AntecedentBehaviorConsequence ABC contingency In operant conditioning a behavior Response occurs in the presence of an antecedent stimulus and the future probability of that behavior occurring under similar antecedent conditions is influenced by its consequences the second Stimulus in SRS or Consequence in ABC The consequence is crucial for the future strength and likelihood of the operant behavior Options A C and D incorrectly describe the fundamental contingencies or misrepresent the sequence

#50. In the context of Applied Behavior Analysis, particularly when establishing a baseline before an intervention, what concept does ‘steady-state responding’ most accurately refer to?

Steadystate responding is a critical concept in experimental design within ABA primarily referring to the stability of a target behavior during baseline measurement The goal is to collect enough data points to observe a consistent pattern of behavior characterized by minimal variability in its level and trend This stability is crucial because it provides a reliable and unbiased reference point or baseline against which the effects of a subsequent intervention can be accurately compared If baseline data is highly variable it becomes exceptionally difficult to ascertain whether any observed changes during intervention are truly due to the intervention itself or merely natural fluctuations in the behavior Options describing rapid correct responses or highprobability requests relate to other behavioral concepts fluencyrate and behavior momentum respectively but do not define steadystate responding

#51. A Board Certified Behavior Analyst (BCBA) is setting up a data collection system to track a fourth-grade client’s ‘remaining seated’ behavior during a 30-minute class, utilizing a whole interval data collection method. The client is known to have difficulty remaining seated for extended periods. Given the goal to increase this behavior, which of the following setups is the most appropriate and accurate for implementing this measurement system?

This question assesses understanding of both the definition of whole interval recording and best practices for interval duration in discontinuous measurement 1 Whole Interval Recording Definition For an interval to be recorded as an occurrence correct the target behavior must occur throughout the entire duration of that interval This method tends to underestimate the actual occurrence of the behavior making it suitable for behaviors targeted for increase Option A is incorrect because some part of the interval describes Partial Interval Recording not Whole Interval 2 Optimal Interval Duration When using discontinuous measurement methods like whole interval shorter intervals are generally preferred especially when the goal is to increase a behavior Shorter intervals eg 1minute intervals as in Option B versus 5minute intervals as in Option C offer several advantages Increased Sensitivity They are more sensitive to subtle changes or occurrences of the behavior More Data Points They yield a larger number of data points within the observation period 30 intervals vs 6 intervals providing a richer dataset for analysis Reduced Underestimation While whole interval always underestimates shorter intervals can help mitigate severe underestimation For instance if a client remains seated for 45 minutes in a 5minute interval it would be recorded as a nonoccurrence In contrast with 1minute intervals those 45 minutes could result in 4 correct recordings providing a more accurate representation of the clients progress Therefore 30 oneminute intervals Option B are superior to 6 fiveminute intervals Option C for this purpose Option D describes Momentary Time Sampling not Whole Interval Recording

#52. A BCBA is developing a task chain for a learner who is just beginning to learn how to ride a bicycle independently. To ensure the task analysis accurately breaks down the complex skill into smaller, teachable steps, the BCBA needs to gather information on how to perform the skill. Considering best practices in Applied Behavior Analysis, which of the following approaches would be the least appropriate to use during the task analysis process?

A task analysis is the process of breaking a complex behavior into smaller teachable units When conducting a task analysis it is crucial to derive the steps from an accurate and efficient performance of the behavior Observing an expert Option A a proficient individual Option B assuming the BCBA is proficient or consulting a specialist Option C are all appropriate methods because these sources can provide a reliable and optimal sequence of steps However observing another child of the same chronological age and skill level as the learner Option D is the least appropriate method This is because a peer who is also learning or struggling with the skill may not perform it correctly efficiently or safely Relying on an inaccurate model would result in a flawed task chain potentially teaching errors or ineffective strategies to the learner making skill acquisition more challenging and potentially reinforcing incorrect behaviors

#53. Dr. Anya, a BCBA, is working with a learner to establish robust relations between printed words, pictures, and the actual objects they represent. Her goal is to ensure the learner understands that ‘CAT’ (written word), an image of a cat, and a live cat all refer to the same concept, even if not directly taught all relations. To effectively assess if stimulus equivalencespecifically reflexivity, symmetry, and transitivityhas been established among these stimuli, which of the following teaching and assessment strategies would Dr. Anya primarily utilize?

The text clearly states Stimulus equivalence is best assessed using what Matching to sample yes It further elaborates with an example if i wanted to assess reflexivity i would give you a picture of a fire truck and you would match that fire truck to the same picture of a fire truck okay and then i would teach you that picture equals the word fire truck and then we would get to the point where you would identify the word fire truck and the picture as a real fire truck so if you can match to sample the word to the picture the picture to a real fire truck we can assess stimulus equivalence Matchingtosample MTS is a cornerstone method for establishing and assessing conditional discriminations which are fundamental to stimulus equivalence It directly tests the learners ability to relate a sample stimulus to a comparison stimulus Shaping involves reinforcing successive approximations of a behavior which is a teaching procedure for developing new behaviors not for assessing equivalence relations Expressive instructions are a method of delivering prompts or commands eg Touch the cat not a primary assessment for derived stimulus relations Focusing on maintenance targets occurs after the initial learning and establishment of equivalence to ensure retention rather than being the primary method for assessing if equivalence has been established

#54. A behavior analyst is working with a client in a clinic setting. To promote generalization of learned skills, the clinic has decided to redecorate each treatment room with different colored furniture, varying decor, and diverse chair styles, ensuring no two rooms look identical. Based on this strategy, what type of generalization intervention is the clinic primarily utilizing?

The clinics strategy of redecorating rooms with different colored furniture varying decor and diverse chair styles is an example of Train Loosely This intervention involves varying noncritical aspects of the instructional environment eg the color of furniture decorations to promote generalization The goal is to make the learners behavior less dependent on specific nonessential environmental cues General Case Procedure would involve systematically identifying and teaching across the full range of stimulus variations and response requirements in the generalization setting which is far more extensive than simply changing decor Multiple Exemplar Training involves teaching the learner to respond to a variety of critical stimuli or to emit a variety of critical responses in different situations While it involves multiple examples it specifically targets critical features or responses not just noncritical environmental changes Indiscriminable Contingency refers to arranging contingencies of reinforcement in such a way that the learner cannot discriminate when reinforcement is available or whether a given response will produce reinforcement This is typically achieved by using intermittent schedules of reinforcement and varying the delivery of reinforcement which is not what is described in the scenario

#55. A Board Certified Behavior Analyst (BCBA) is conducting a functional analysis to determine if a client’s problem behavior is maintained by access to tangible items. Based on the procedural description provided in the text, which of the following accurately describes the key elements and manipulation for setting up a tangible condition during this functional analysis?

The text explicitly details the setup for a tangible condition in a functional analysis were going to give access to only the preferred tangible adult attention and neutral or moderately preferred toys and then were going to remove the preferred tangible occasionally This precise configuration is crucial It ensures that the client has initial access to the presumed reinforcer preferred tangible along with typical environmental components like adult attention and other nontarget items before the tangible is withdrawn to evoke the problem behavior This systematic manipulation allows the analyst to determine if obtaining the tangible is indeed the function of the behavior Option A is incorrect because it omits adult attention and other toys and suggests contingent removal which is not how the baseline access is established or removed occasionally to test the condition as described Option C inaccurately describes the initial access and states removal only upon behavior which is a specific form of contingent presentation not the general tangible condition described Option D is incorrect as the tangible must be removed at times to test its function and other highly preferred items would confound the specific tangible being tested

#56. A behavior analyst who owns and operates her own ABA company is committed to continuously improving the quality of service provided by her Registered Behavior Technicians (RBTs) working in the field. She understands that effective supervision and performance management for RBTs should mirror the empirically supported principles of behavior change used with clients. Considering this, which of the following strategies would be most likely to be the most effective for enhancing the performance of her RBTs?

The most effective strategy for improving RBT performance mirrors the principles of effective behaviorchange programs for clients direct observation individualization and datadriven decisionmaking Lets analyze why option C is the most effective and why others are less so Option C Direct observation individualized goals performancebased feedback This strategy embodies the core principles of ABA when applied to personnel management Just as we conduct direct assessments and create individualized treatment plans for clients supervisors should directly observe their RBTs in their natural work environment insitu to accurately assess performance This direct observation provides objective data which then informs the creation of individualized performance goals Collaborative goal setting and performancebased feedback ensure that interventions are tailored to the specific needs of each technician maximizing their effectiveness and buyin This approach leads to true skill development and performance improvement Option A Universal incentive program based on overall client satisfaction While incentives can be motivating a universal incentive based on an aggregate measure like overall client satisfaction has several drawbacks It does not account for individual RBT performance or effort may not function as a reinforcer for all RBTs lack of preference assessment and client satisfaction can be influenced by many factors beyond an individual RBTs control making the contingency indiscernible or unfair Option B Standardized goals and curriculum for all RBTs This approach lacks individualization which is a cornerstone of effective ABA Just as we wouldnt use one treatment plan for all clients a single set of goals or a generic curriculum will not address the diverse skill sets learning styles and performance needs of individual RBTs Some RBTs might already master the skills others might need more intensive support in specific areas making a onesizefitsall approach inefficient and potentially demotivating Option D Selfevaluation and peer feedback Selfevaluations and peer feedback are forms of indirect assessment While they can be supplementary tools relying primarily on them to adjust goals is problematic Selfreports can be biased or inaccurate and peer feedback might lack objectivity or consistency Direct observation by a qualified supervisor is crucial for objective reliable performance data necessary for meaningful goal adjustments and effective supervision

#57. In the field of Applied Behavior Analysis (ABA), particularly within the framework of B.F. Skinner’s analysis of verbal behavior, what is the most crucial aspect that practitioners are primarily focused on when analyzing an individual’s verbal operants?

This question addresses a fundamental principle that underlies all of ABA the emphasis on understanding the function of behavior which is equally applicable to verbal behavior as it is to nonverbal behavior Option A Correct In ABA the core focus is consistently on the function of behavior that is why the behavior occurs This involves identifying the specific environmental variables antecedents and consequences that control and maintain the behavior For verbal behavior this means understanding what evokes a verbal response and what the consequence of that response is For example saying water can function as a mand a request for water evoked by thirst and reinforced by receiving water a tact labeling water when shown a picture of it evoked by the picture and reinforced by generalized praise or an intraverbal saying water when asked What do you drink evoked by the verbal prompt and reinforced by generalized praise Understanding the function is paramount because it dictates how to effectively teach and modify verbal skills Option B Incorrect While the form or topography how the behavior looks or sounds eg the specific words used articulation or sentence structure is observable and can be measured it is not the primary focus in ABA A behaviors form can be identical yet its function can be vastly different depending on the controlling variables For instance a child pointing to a cookie form could be manding requesting or tacting labeling based on the context Option C Incorrect Magnitude or intensity eg speech volume or loudness is a dimension of behavior that can be quantified and may be a target for intervention eg teaching a child to speak louder However like form magnitude describes how the behavior occurs not why it occurs It is a characteristic of the behaviors topography rather than its underlying function Option D Incorrect The rate of verbal behavior frequency per unit of time is a critical measure used to track progress and evaluate the effectiveness of interventions While essential for data collection and analysis it quantifies how often a behavior occurs not why it occurs The function of a behavior explains the controlling variables that in turn influence its rate

#58. A Board Certified Behavior Analyst (BCBA) has been working with an adult client on an intervention designed to increase their social engagement in community settings. The client has made significant progress, demonstrating an increase in initiation of conversations and participation in group activities. Despite this objective progress, the client informs the BCBA that they no longer wish to continue with the intervention, stating they feel they have achieved their personal goals for social interaction and prefer to focus on other aspects of their life. The BCBA believes more progress could be made. According to ethical guidelines, what is the most appropriate action for the BCBA to take in this scenario?

This scenario directly addresses the ethical responsibility of a BCBA to respect client autonomy and the right to selfdetermination as outlined in the Professional and Ethical Compliance Code for Behavior Analysts Even if the BCBA believes further progress is possible or desirable an adult client who is of sound mind has the right to make decisions about their own services including termination Option C correctly emphasizes encouraging the client by showing progress which is a supportive and ethical approach while also respecting their decision Option A is incorrect because client rights supersede adherence to program guidelines when the client wishes to terminate especially if they are not harming themselves or others Option B is unethical as offering an even bigger reward to persuade someone to continue an intervention they wish to stop borders on coercion or bribery which undermines voluntary participation and client dignity Option D while potentially true that some regression could occur is an attempt to influence the clients decision through potential negative consequences which again disrespects their autonomy and right to choose The core principle is that clients are not obligated to continue services they no longer desire and the BCBAs role is to support not control the clients life choices

#59. Three sorority sisters, one a BCBA and another a marketing consultant, graduated from their master’s programs together. The marketing consultant’s son has special needs, and she proposes exchanging marketing tips for the BCBA’s behavior analytic services for her son. Considering the BACB’s Ethical Code for Behavior Analysts, is this arrangement permissible, and if so, under what primary condition?

The BACBs Ethical Code permits bartering for services but it is not always acceptable The crucial stipulation is that the value of the goods or services exchanged must be approximately equal to the value of the behavior analytic services This helps prevent exploitation perceived conflicts of interest and ensures fair compensation While the preexisting relationship sorority sisters introduces a potential for a dual relationship the primary concern directly addressed in the text regarding this specific arrangement bartering is the equality of value If the values are not equal it creates an ethical dilemma regardless of the preexisting relationship Referring the client elsewhere is an option to avoid dual relationships but the prompt asks if the arrangement is okay and under what condition implying that it can be okay if stipulations are met Therefore the core condition for permissible bartering as explained in the text is the equality of value

#60. A Board Certified Behavior Analyst (BCBA) is working with a client who frequently engages in repetitive hand-flapping. The BCBA is primarily concerned with understanding the onset of these behaviors in relation to specific environmental triggers. Therefore, the BCBA decides to record the time from the presentation of a trigger (e.g., a novel stimulus) to the initiation of hand-flapping. Which fundamental dimensional quantity of behavior is the BCBA primarily focused on measuring in this scenario?

Temporal Locus is one of the three fundamental dimensional quantities of behavior along with repeatability and temporal extent It refers to the point in time at which a behavior occurs Derivatives of temporal locus include latency the time from the onset of a stimulus to the initiation of a response which is precisely what the BCBA is measuring here and interresponse time IRT the time between two consecutive responses Repeatability refers to the fact that a behavior can be counted or occurs repeatedly over time eg frequency count rate Temporal extent refers to the duration of a behavior how long it lasts Rate is a derivative of repeatability specifically count per unit of time not a fundamental dimensional quantity itself

#61. A behavior analyst is conducting a research study to evaluate the effectiveness of a new social skills intervention for adolescents with autism spectrum disorder. The analyst plans to systematically introduce, withdraw, and then reintroduce components of the intervention across different phases of the study to observe their impact on the participants’ social interaction behaviors. When the behavior analyst is engaging in the systematic process of adding, removing, or modifying these intervention components within the experimental design, what specific type of variable is the analyst manipulating to determine its effect on the target behavior?

In experimental design within Applied Behavior Analysis the independent variable IV is the environmental event or condition that the researcher systematically manipulates introduces or withdraws to determine its effect on the behavior of interest In this scenario the intervention components eg specific social skills teaching strategies prompts reinforcement contingencies are precisely what the behavior analyst is manipulating adding removing modifying The purpose of these manipulations is to observe whether changes in these interventions cause changes in the target behavior The dependent variable DV is the behavior being measured eg social interaction behaviors Confounding variables are uncontrolled factors that might inadvertently influence the dependent variable potentially obscuring the true effect of the IV Extraneous variables are all other environmental variables not directly controlled or measured in the experiment

#62. A behavior analyst has a morning routine of making tea for their family. In the afternoon, they offer more tea, but their family consistently responds with a verbal ‘no’ three times. After these three instances, the behavior analyst observes that their own behavior of offering tea in the afternoon significantly decreases in future frequency. Which of the following behavior-change procedures best describes what occurred to the behavior analyst’s afternoon tea-offering behavior?

The behavior analysts behavior of offering tea decreased A decrease in the future probability of a behavior indicates either a punishment procedure or an extinction procedure Extinction involves withholding reinforcement from a behavior that was previously reinforced In this scenario the behavior of offering tea was not described as having been previously reinforced rather the verbal response no was added to the environment immediately following the behavior which then led to a decrease in the future likelihood of the teaoffering behavior Since a stimulus no was presented added and the behavior decreased this is a clear example of positive punishment Negative punishment would involve the removal of a stimulus and negative reinforcement would involve the removal of an aversive stimulus leading to an increase in behavior

#63. When a punishment procedure is deemed necessary and ethically justified to rapidly decrease a severe problem behavior, what principle regarding the intensity of the punishing stimulus should guide its initial implementation?

The text explicitly states punishment should always be what it should always be the most intense you can get away with from the beginning we never fade in punishment that should always be the Peak from the start This principle is crucial in ABA When punishment is necessary it should be implemented at an effective intensity from the beginning to achieve rapid and lasting suppression of the target behavior Fading in punishment can lead to habituation where the individual adapts to milder levels making higher intensities necessary later and potentially prolonging the exposure to aversive stimuli The goal is to make the intervention as brief and effective as possible minimizing overall exposure to the punisher while maximizing its impact on the target behavior Ethical considerations always guide the decision to use punishment ensuring it is least restrictive effective and accompanied by reinforcement for alternative behaviors

#64. When you first moved into an apartment with your new roommate, you noticed that your roommate would reliably sneeze every time the front door was opened, seemingly due to a dust allergy exacerbated by the draft. Over the course of a month, after you frequently opened the front door in front of your roommate, they now rarely sneeze when the door is opened. This phenomenon is best described as an example of what behavioral process?

Sneezing is a reflex which is a respondent behavior meaning it is elicited by an antecedent stimulus in this case the opening door which was likely conditioned to elicit sneezing due to its association with dustdraft Respondent Extinction occurs when a conditioned stimulus the opening door is repeatedly presented without the unconditioned stimulus the full force of the allergen or irritant or perhaps the association weakens leading to a gradual decrease and eventual elimination of the conditioned response sneezing The text also refers to this as habituation which is a related phenomenon where there is a decrease in the strength of an elicited behavior due to repeated presentation of the eliciting stimulus Lets clarify the other options Operant Extinction applies to operant behaviors where a behavior decreases because it no longer produces reinforcement Sneezing is a respondent behavior not an operant one maintained by consequences in this context Habilitation refers to the degree to which a persons repertoire maximizes short and longterm reinforcers and minimizes punishers essentially describing the overall success and adjustment of an individual in their environment It is an outcome measure not a specific behavioral process Reversal Design or ABAB design is an experimental design used in research to demonstrate functional control between an intervention and a behavior not a natural behavioral process occurring within an individual

#65. A client engages in frequent tantrum behaviors, which a functional assessment has determined are maintained by socially mediated tangible reinforcement (e.g., access to preferred toys or snacks provided by an adult). The behavior analyst wants to develop an antecedent intervention to manipulate the Motivating Operation (MO) for this specific reinforcer, thereby decreasing the likelihood of tantrums. Which of the following interventions would be the most appropriate strategy for the analyst to implement?

The question specifically asks for an antecedent intervention to manipulate the Motivating Operation MO for socially mediated tangible reinforcement The text explains that to manipulate an MO for reinforcement one should think of deprivation and satiation Option C describes NonContingent Reinforcement NCR with tangibles By providing noncontingent ie not dependent on behavior access to preferred tangibles on a timebased schedule the MO for tangible items is abolished satiation occurs This makes the client less motivated to engage in tantrum behavior to gain those tangibles because they are already freely available Option A addresses an escape function by following through with a demand the client escapes the demand if the behavior was escapemaintained Option B describes NCR for attention which would be appropriate if the tantrums were attentionmaintained not tangiblemaintained Option D describes a punishment procedure which is a consequencebased intervention not an antecedent manipulation of the MO and is generally not the first line of intervention

#66. Greg’s mother used a forward chaining procedure to teach him how to mop. During the initial teaching phase, to ensure early success and focus on later steps, the mop was always placed out, and the mop bucket was already filled by his mother before Greg began. Now that Greg has mastered the entire mopping sequence that was initially taught (from taking the mop to putting it away), his mother has stopped filling the mop bucket herself, requiring Greg to perform that step. What principle is Greg’s mother applying by no longer pre-filling the mop bucket and expecting Greg to do it?

In a behavioral chain all steps must eventually be performed independently by the learner to complete the task When Gregs mother initially taught the mopping sequence she consistently performed the fill the mop bucket step for him This means that step was omitted from Gregs direct instruction within the forward chain he was never taught to perform it himself Now that Greg has mastered the previously taught sequence his mother is introducing this omitted step making him responsible for its execution This strategy ensures the learner acquires all necessary components of the task moving towards full independence for the entire behavioral chain While it might involve elements of fading the mothers presence as a prompt the core action is the addition and instruction of a step that was previously done for the learner rather than fading a prompt that was explicitly given for Greg to perform that specific step

#67. You are a Board Certified Behavior Analyst (BCBA) supervising a student accruing supervised independent fieldwork hours. The student approaches you at the end of May with supervision documentation sheets for February and March, requesting your signature for those past months. What is your ethical obligation in this situation according to the BACB guidelines?

The BACBs ethical guidelines and administrative requirements for supervision are strict regarding documentation Supervision sheets and other documentation must be signed and submitted within the designated reporting period typically the month in which the supervision occurred Backdating supervision sheets is explicitly prohibited as it can misrepresent the timeline of supervision and compromise the integrity of the certification process Therefore the BCBAs ethical obligation is to refuse to sign the backdated sheets While the hours for those months would not count this action upholds the integrity of the profession and adherence to the BACBs rules Removing the student from the program altogether is a more severe disciplinary action not directly implied by this specific scenario though the hours in question would be invalid

#68. A behavior analyst is discussing the properties of verbal operants with a new supervisee, specifically highlighting the differences between point-to-point correspondence and formal similarity. The analyst presents several scenarios and asks the supervisee to identify which one definitively demonstrates point-to-point correspondence. Which of the following examples should the supervisee identify as representing point-to-point correspondence?

Pointtopoint correspondence is a property of a verbal operant where the beginning middle and end of the verbal stimulus matches the beginning middle and end of the verbal response This means that parts of the stimulus control parts of the response and the entire response product is identical to the stimulus In the given options A Seeing a rabbit and saying rabbit lacks pointtopoint correspondence because the stimulus is a physical object and the response is a sound they are not identical This is an example of a tact B When you write down the phone number exactly as it was texted to you there is pointtopoint correspondence The textual stimulus the texted number and the written response the number on paper are identical in their sequence of elements While the forms texted vs written are different the content has pointtopoint correspondence If the forms were the same eg writing down a number you read from a written sign it would also have formal similarity C The cashiers question and your reply involve spoken language formal similarity but the responses are not identical to the stimuli thus lacking pointtopoint correspondence This is an example of an intraverbal D Reading a Stop sign aloud and saying Stop demonstrates pointtopoint correspondence the letters in Stop evoke the sounds Stop This is a textual operant However option B is also a strong example of pointtopoint correspondence even if it lacks formal similarity texted form vs written form

#69. John wants to stay out late on Saturday night. He has learned from past experiences that if he asks his mom for permission, she is likely to say yes, but if he asks his dad, his dad will almost always say no. Consequently, John consistently approaches his mom when he wants to ask to stay out late. In this scenario, how is John’s mom functioning relative to his request to stay out late?

In Applied Behavior Analysis a discriminative stimulus SD is a stimulus in the presence of which a particular operant behavior is more likely to be reinforced In Johns scenario his moms presence or the act of asking her has through a history of reinforcement her saying yes become a signal that reinforcement permission to stay out is available for the behavior of asking to stay out Conversely his dads presence functions as an Sdelta S a stimulus in the presence of which a particular operant behavior is not reinforced John has learned to discriminate between the two parents based on the differential consequences his request has historically received An establishing operation EO would alter the value of the reinforcement eg making staying out more desirable or the frequency of behaviors that have been reinforced by that consequence but the mom herself is not changing the value of staying out but rather signaling the availability of the reinforcer A discriminative stimulus for punishment SDp would signal that a behavior is likely to be punished An abolishing operation AO would decrease the effectiveness of a reinforcer which is also not the case here Therefore Johns mom is functioning as a discriminative stimulus for reinforcement

#70. In Applied Behavior Analysis, stimuli are often categorized into different classes based on their shared features or functions. Which of the following options is NOT considered a type of stimulus class?

Stimulus classes categorize stimuli based on their properties or effects on behavior A formal stimulus class refers to stimuli that share common physical properties eg shape color size A temporal stimulus class refers to stimuli that occur at a specific time relative to the behavior eg antecedents consequences A functional stimulus class refers to stimuli that evoke the same response or serve the same function eg all stimuli that signal the availability of reinforcement for a particular behavior Generalizable however is not a type of stimulus class itself Instead generalization refers to a behavioral process where a learned response occurs in the presence of stimuli that are similar to but not identical to the trained stimulus While we discuss stimulus generalization it describes a phenomenon of behavior across stimuli not a classification of the stimuli themselves

#71. A steakhouse hires a new head chef who wishes to thoroughly understand the physical layout of the kitchen and the specific types and locations of appliances available before commencing work. From an ABA perspective, akin to a behavior analyst familiarizing themselves with a new client’s environment, what is the most appropriate assessment method for the chef to utilize to achieve this understanding?

An ecological assessment involves a comprehensive evaluation of the clients environment including physical features social interactions past learning history and other environmental conditions that may affect behavior The goal is to identify and describe environmental factors that might be related to the behavior of interest In this scenario the chefs need to familiarize themselves with the layout of the kitchen and the appliances directly aligns with the purpose of an ecological assessment which focuses on understanding the environment in which behavior occurs This direct observation and analysis of the physical setting the kitchen is crucial for the chef to understand the context of their work just as it is for a behavior analyst to understand a clients home school or community environment While other options interviewing staff meetings might provide useful indirect information or serve other supervisory purposes they do not directly achieve the goal of familiarizing oneself with the environment itself in the same comprehensive direct manner as an ecological assessment

#72. Two Board Certified Behavior Analysts (BCBAs) at a clinic are planning a research study to investigate the effectiveness of a novel play-based instructional approach on the social-communicative behaviors of a group of four-year-old children diagnosed with autism spectrum disorder. Before the BCBAs can begin any data collection or intervention implementation with the children, what fundamental ethical requirement must they fulfill, as specifically mentioned in the provided text regarding research involvement?

The text directly states The parents give informed consent and agree when discussing behavior analysts conducting research This highlights informed consent as a critical and nonnegotiable ethical requirement in all research involving human participants Informed consent ensures that participants or their legal guardians in the case of minors are fully aware of the nature of the research including its purpose procedures potential risks benefits and their right to withdraw at any time before voluntarily agreeing to participate This principle is a cornerstone of ethical research practice in Applied Behavior Analysis BACB Professional and Ethical Compliance Code for Behavior Analysts Section 90 Public Statements and Research While securing funding conducting functional analyses for clinical intervention not always for all research types or educational qualifications are important aspects of research informed consent is the specific ethical prerequisite for engaging participants mentioned in the text

#73. Jay implements an intervention to teach a client independent dressing skills. The client begins dressing themselves independently, which is a positive outcome. However, Jay later discovers that the client’s parents were also working on dressing at home, and the client may have watched instructional videos online. Based on this information, what critical aspect does Jay’s intervention most likely lack?

Internal validity refers to the extent to which an experiment demonstrates that the independent variable the intervention in this case Jays teaching is solely responsible for the observed changes in the dependent variable the clients independent dressing and not other extraneous confounding factors In Jays scenario the presence of multiple potential influences on the clients behavior parents teaching YouTube videos makes it difficult to definitively conclude that Jays intervention alone caused the observed improvement This lack of certainty regarding the causal relationship between the intervention and the behavior change indicates a lack of internal validity External validity by contrast refers to the generalizability of research findings to other settings populations or conditions which is not the primary issue here Social validity pertains to the social significance of the target behavior the appropriateness of the intervention procedures and the overall impact of the behavior change independent dressing is typically a socially valid skill Observer validity often referring to interobserver agreement or IOA relates to the consistency and accuracy of data collection among different observers which is a measurement concern not a concern about the source of behavior change itself

#74. A Board Certified Behavior Analyst (BCBA) is collaborating with a speech therapist on a client’s intervention plan. During a team meeting, the speech therapist proposes an intervention strategy that the BCBA is not entirely familiar with and has some reservations about its immediate efficacy from an ABA perspective. To ensure client welfare, foster interdisciplinary collaboration, and adhere to ethical guidelines, what is the most appropriate initial course of action for the BCBA?

This question addresses the critical ethical and professional responsibility of interdisciplinary collaboration and datadriven decisionmaking in ABA Option D represents the most ethical and effective approach By agreeing to implement the intervention and collecting data the BCBA demonstrates a commitment to collaboration and client welfare This allows for an empirical evaluation of the interventions effectiveness If the data show positive results its a win for the client and the team If the data indicate the intervention is not effective the BCBA has objective information to discuss with the speech therapist and stakeholders allowing for informed modifications or alternative strategies Options A and C are counterproductive to collaboration and could negatively impact client services Option B is unethical as it undermines a team member behind their back and erodes trust among professionals and with the family

#75. A behavior analyst is collecting duration data to assess a client’s engagement in prosocial peer interaction within a structured social skills group. During a 60-minute observation period, the analyst recorded the following instances of the client talking to peers 5 minutes, 10 minutes, 8 minutes, and 12 minutes. Given this data, what percentage of the total observation hour did the client spend talking to peers?

To determine the percentage of time the client spent talking to peers first sum the individual durations recorded 5 minutes 10 minutes 8 minutes 12 minutes 35 minutes Next divide this total duration by the entire observation period which is 1 hour or 60 minutes 35 minutes 60 minutes 05833 Finally multiply by 100 to convert this decimal to a percentage 05833 100 5833 Rounded to the nearest whole number the client spent 58 of the first hour talking to peers This calculation is a fundamental skill for interpreting duration data and communicating behavioral trends effectively

#76. Blake typically has his iPad removed (a negative punishment procedure) if he plays for more than one hour continuously without stopping. This procedure has been highly effective in the past, consistently reducing his screen time. However, this week, Blake has been ill, and his parents, due to his poor health, have not consistently implemented the iPad removal contingency. As a result, Blake has started playing on his iPad for two to three hours at a time without it being taken away, and his screen time has returned to pre-punishment levels. Given that the punishment procedure has ceased to be applied, what behavioral phenomenon is Blake currently exhibiting?

Blake is exhibiting recovery from punishment Recovery from punishment is a behavioral phenomenon where when a previously effective punishment procedure is terminated or no longer consistently applied the punished behavior tends to return to its original or even higher prepunishment levels The text clearly states that the punishment iPad removal was effective but is now no longer being consistently implemented leading to the resurgence of the iPad playing behavior Lets analyze why the other options are incorrect Spontaneous recovery Spontaneous recovery is a phenomenon associated with extinction It occurs when a previously extinguished behavior a behavior that has decreased due to the withholding of reinforcement temporarily reemerges after a period of time during which the behavior has not been engaged in Since the scenario describes the cessation of a punishment procedure not an extinction procedure spontaneous recovery is not the correct term Resistance to punishment Resistance to punishment refers to a situation where a behavior does not decrease or is only minimally affected by the application of a punishment procedure In Blakes case the punishment was initially effective at reducing his screen time so he was not resisting it The current situation is about the absence of punishment not his resistance to it Negative reinforcement Negative reinforcement involves the removal of an aversive stimulus contingent on a behavior which increases the future probability of that behavior This scenario describes the cessation of a punishment procedure which led to the return of a behavior not the increase of a new behavior due to the removal of an aversive stimulus

#77. A three-year-old child frequently repeats what he hears at home and on television. Yesterday, while at the park, he pointed to various objects he saw, such as a swing and a dog, and said ‘Swing’ and ‘Dog’ respectively. Based on his behavior specifically at the park, which verbal operant is the three-year-old primarily engaging in?

The text highlights that the key to this question is focusing on the behavior at the park While the child may engage in echoic behavior at home by repeating what he hears his behavior at the park is described as he started saying out loud things he saw while walking at the park This is further clarified as hes essentially labeling things he sees while walking A tact is a verbal operant where a nonverbal discriminative stimulus eg seeing a swing or a dog evokes a verbal response The text explicitly states all hes doing is seeing something labeling it nonverbal SD evoking verbal Behavior he is tactic hes simply labeling it It is not an echoic because he is not repeating what he heard at the park It is not a mand because he is not making a request It is not an intraverbal because he is not responding to a verbal stimulus with a verbal response that has no pointtopoint correspondence with the stimulus instead he is labeling something he visually perceives

#78. Your best friend excitedly tells you a detailed story about witnessing a truly incredible and rare event, describing it with specific times, locations, and vivid details. After listening intently, you respond by saying, ‘That’s an amazing story, and I appreciate the detail, but I’ll believe it when I see it with my own eyes or am presented with concrete, verifiable evidence.’ This response most closely reflects adherence to which attitude of science?

Empiricism is the objective observation of the phenomena of interest It is the practice of objective observation and measurement free from bias The statement Ill believe it when I see it with my own eyes or am presented with concrete verifiable evidence directly aligns with empiricism as it emphasizes the fundamental need for direct observable and measurable evidence to accept a claim as true This attitude underscores the importance of data and observable events over personal anecdotes or assumptions Parsimony states that one should choose the simplest most logical explanation that adequately accounts for the observed phenomenon Determinism is the philosophical assumption that the universe is a lawful and orderly place and that all phenomena occur as a result of other events While philosophical doubt involves questioning the truthfulness of scientific findings empiricism is the more fundamental attitude demanding observable evidence for belief which is precisely what the person is asking for in this scenario

#79. A Board Certified Behavior Analyst (BCBA) is reviewing data collected by a newly hired Registered Behavior Technician (RBT). The BCBA observes that the RBT’s method of measuring a target behavior consistently produces data that is inaccurate. The RBT is not naive to the purpose of the measurement or the target behavior. According to ABA principles, what term best describes the inaccurate data resulting from the way the RBT measured the behavior?

An artifact in the context of data measurement refers to data that is inaccurate due to the specific method by which it was measured The provided text explicitly states when we get data as a result of a way we measured it and its inaccurate so the way we measured it led to inaccurate data we consider this an artifact This phenomenon occurs when the measurement process itself rather than the true occurrence of the behavior distorts the data A naive observers bias is incorrect because the text clarifies that the rbt is not a naive observer they understand what theyre measuring what the goal is what theyre trying to accomplish Therefore the inaccuracy is not due to a lack of understanding by the observer A confound is incorrect The text differentiates an artifact from a confound explaining that a confound typically is found in an experimental setting thats when an outside variable affects our dependent variable In this scenario the issue stems directly from the measurement method itself leading to inaccurate data rather than an uncontrolled external variable influencing the target behavior Unreliable data is not the most precise or best answer in this context While inaccurate data can certainly lead to unreliable data lack of consistency across repeated measurements reliability specifically refers to the consistency of measurement across repeated observations we can repeatedly measure the same thing over and over again The core problem described is the inaccuracy that directly stems from the measurement method which is precisely what an artifact describes The text highlights that just because the rbt is inaccurate doesnt mean theyre unreliable implying that inaccuracy from the method an artifact is the more direct and certain description of the discrepancy in this specific scenario

#80. A Board Certified Assistant Behavior Analyst (BCaBA) working under the supervision of a BCBA observes a professional from a different discipline (e.g., a physical therapist) implementing an intervention for a shared client that, in the BCaBA’s professional judgment, appears to be based on an unvalidated technique and potentially contradicts some core principles of applied behavior analysis. The BCaBA reports this concern to their supervising BCBA. The BCBA confirms the practice is not aligned with ABA best practices but notes that the professional in question is not a behavior analyst. Considering the BACB’s ethical guidelines and the information provided in the text, what is a crucial point for the BCBA to remember and communicate in this situation?

The text explicitly clarifies the ethical code only applies to the ABA practitioners so if youre contracted youre working with other people they dont have to adhere to our our code remember that right you see somebody else doing something but theyre not a behavior analyst well oh well right you can still talk to him about it but they have no obligation to our code This means that while it is good practice for the BCBA to collaborate and educate they cannot enforce the BACBs ethical code on professionals from other disciplines Options A C and D are incorrect because they misinterpret the scope and applicability of the BACBs ethical code implying an authority that does not exist over nonABA professionals

#81. Maryland residents are participating in an election by mailing in their ballots. A team of individuals is responsible for collecting and counting these ballots. From the perspective of the people counting the ballots, what type of measurement system would accurately describe the data they are primarily concerned with when determining voting outcomes?

This question focuses on identifying the correct measurement procedure when the behavior itself is not directly observed Permanent product measurement involves observing the lasting effects or tangible outcomes of a behavior rather than observing the behavior as it occurs In this scenario the individuals counting the ballots are not directly observing people as they vote or mail their ballots Instead they are analyzing the completed ballots which are the direct and enduring results permanent products of the voters behavior The ballot itself serves as the observable outcome Option A Rate would require observing the voting behavior over time which the counters are not doing Option B ABC recording involves observing the environmental context of a behavior and its consequences which is not the primary task of ballot counters Option D PLACHECK is a form of momentary time sampling used to observe a groups engagement in an activity which is also not applicable here Therefore permanent product is the most accurate description of the measurement approach from the perspective of the ballot counters

#82. Sam, a newly certified BCBA, is working with a client who exhibits attention-seeking behaviors. To understand the environmental variables influencing these behaviors, Sam intentionally removes attention from the client, observes the subsequent increase in attention-seeking behaviors, and then provides attention contingent on a desired alternative behavior. Which of the following best describes the assessment strategy Sam is employing?

A functional analysis FA is a direct assessment method that involves systematically manipulating antecedents andor consequences to determine their effect on the target behavior By depriving the client of attention an antecedent manipulation to evoke attentionseeking behavior and then providing attention for a desired behavior Sam is directly testing hypotheses about the function of the behavior in a controlled manner This systematic manipulation allows for a more definitive identification of the maintaining variables function of the behavior compared to other assessment methods Indirect assessment gathers information through interviews or checklists without direct manipulation or observation Implementing a punishment procedure aims to decrease behavior by adding an aversive stimulus or removing a desirable one which is not the primary purpose of an FA though an FA may involve consequences A simple antecedent intervention modifies antecedents to prevent behavior but does not involve the systematic manipulation for functional identification that characterizes an FA

#83. A Board Certified Behavior Analyst (BCBA) is developing an intervention plan for a client who engages in attention-seeking challenging behaviors. The BCBA hypothesizes that the challenging behaviors are maintained by attention. To reduce the occurrence of these behaviors, the BCBA decides to implement a strategy where attention is delivered to the client on a fixed-time schedule (e.g., every 5 minutes) regardless of whether the challenging behavior is occurring. This intervention aims to decrease the client’s desire for attention, thereby reducing the motivation for challenging behaviors. Which of the following best describes the intervention being used and the principle it directly manipulates?

Noncontingent reinforcement NCR involves delivering a reinforcer on a fixedtime FT or variabletime VT schedule independent of the clients behavior By frequently providing the maintaining reinforcer attention in this case without requiring the challenging behavior NCR works to decrease the value of that reinforcer thereby functioning as an abolishing operation AO This reduces the motivating operation MO for the challenging behavior making the behavior less likely to occur because the consequence it previously produced is no longer as valuable or desired The provided text explicitly states NCR is a way to manipulate MOs specifically to decrease the value of that attention thus manipulating dmo Positive and negative reinforcement are consequences that follow behavior and increase or decrease its future likelihood but they do not primarily manipulate MOs in this direct manner DRO is a different procedure involving withholding reinforcement for the target behavior and delivering it for the absence of the behavior or for any other behavior not primarily an MO manipulation technique in this context

#84. A Board Certified Behavior Analyst (BCBA) is working with a school for gifted children to increase their use of complex words and sentences. They have designed an intervention and are currently collecting baseline data. The BCBA should not recommend initiating the intervention if the baseline data indicate which of the following?

When the goal is to increase a specific behavior and the baseline data already demonstrates a clear increasing trend it suggests that the behavior may be improving or developing without the immediate need for a formal intervention In such circumstances the BCBA might choose to continue monitoring the behavior to determine if it can reach the desired terminal goal on its own Initiating an intervention when the behavior is already trending in the desired direction could make it difficult to ascertain if the intervention itself or other natural variables caused the observed improvement Conversely if the baseline is stable or decreasing intervention is generally warranted to achieve the desired increase High variability might indicate a need for more baseline data collection or further analysis to identify controlling variables before making a decision but an increasing trend for a goal to increase is the most direct reason not to start an intervention immediately

#85. Susanna folds 15 articles of clothing per minute. As her BCBA, you are interested in determining her average clothes-folding response time (RT) per article. Based on this information, what is Susanna’s average clothes-folding RT in seconds per article?

To calculate the average response time RT per article we need to divide the total time by the number of articles folded The problem states Susanna folds 15 articles per minute A minute consists of 60 seconds Therefore the calculation is 60 seconds 15 articles 4 seconds per article This means on average it takes Susanna 4 seconds to fold one article of clothing The text explicitly demonstrates this calculation 60 divided by 15 you get 4 seconds So what is Susannas average close voting RT Its going to be 4 seconds

#86. Matt, an adult with an intellectual disability, works as a cashier at a local diner. He is very friendly but sometimes greets customers more than once during a single interaction, which his boss wants to reduce. To address this, Matt’s boss tells him that if he greets customers only once per interaction, he will receive a small bonus on his paycheck. What type of differential reinforcement intervention is Matt’s boss using?

Differential Reinforcement of Diminishing Rates DRD is an intervention used to decrease the frequency of a target behavior Reinforcement is delivered when the rate of the target behavior is at or below a predetermined criterion during a specific interval The goal is to gradually reduce the behaviors occurrence In this scenario Matts boss is reinforcing him specifically for greeting customers only once per interaction which is a specific lower rate than his previous tendency to greet them multiple times The intervention directly targets diminishing the rate of greetings to a desired lower frequency DRO Differential Reinforcement of Other Behavior involves reinforcing the absence of the target behavior for a specified period eg reinforcing Matt if he doesnt greet customers at all for 5 minutes DRA Differential Reinforcement of Alternative Behavior involves reinforcing a desirable alternative behavior while withholding reinforcement for the unwanted behavior While greeting once could be considered an alternative to greeting multiple times the primary focus of the intervention as described is on the rate of the greeting behavior itself making DRD a more precise fit DRI Differential Reinforcement of Incompatible Behavior is a subset of DRA where the alternative behavior is physically impossible to perform simultaneously with the unwanted behavior eg reinforcing Matt for holding a menu while he is also greeting if holding the menu prevented multiple greetings

#87. Philip is learning Russian vocabulary. First, he watches a YouTube video and repeats a Russian word (Stimulus A). Next, he successfully matches the spoken Russian word (A) to a picture of lunch items (Stimulus B), and later, he can identify the picture of lunch items (B) when hearing the Russian word (A). Tomorrow, Philip plans to use the Russian word (A) to verbally order his lunch (Response C) in front of Natalia, demonstrating that the spoken word (A) is functionally equivalent to ordering the food (C) based on its prior relationship with the picture (B). If Philip successfully uses the Russian word to order his lunch, what phenomenon of stimulus equivalence would he have demonstrated?

This question describes a classic example of transitivity within stimulus equivalence Reflexivity is AA matching a stimulus to itself Symmetry is AB and BA the reversibility of a learned relationship as seen when Philip can match the word to the picture and the picture to the word Transitivity however involves three arbitrary stimuli A B C where A is trained with B AB and B is trained with C BC and as a result a novel untaught relationship emerges between A and C AC and often C and A CA In Philips scenario A Russian word B Picture of lunch items was trainedestablished B Picture of lunch items C Actual lunch itemordering food via picture can be inferred or is a known association The novel untaught relationship of A Russian word C Ordering actual food is the transitivity being tested Selfmonitoring is a selfmanagement technique and not part of stimulus equivalence

#88. A BCBA has been contracted by a manufacturing company to address a decline in employee productivity and an increase in safety incidents on the factory floor. Before developing any specific interventions for individual employees or groups, the BCBA decides that the most critical first step is to gain a comprehensive understanding of the entire work environment. This includes observing the physical layout, common work routines, typical noise levels, social interactions among shifts, existing communication channels, and the general company culture. Which type of assessment best aligns with the BCBA’s initial goal of familiarizing herself with the broader context and associated environment of the workplace?

An ecological assessment is the most appropriate and comprehensive choice when the primary goal is to gain a thorough understanding of the entire environment surrounding the individuals or group as described in the scenario The text explicitly defines an ecological assessment as one that looks at the surrounding environment of the learner or in this case the staff It is a specific type of direct assessment that systematically considers all relevant environmental variables eg physical setting social context routines historical data that might influence behavior While direct observation C is a crucial component and certainly necessary to gather data within an ecological assessment an ecological assessment is a more encompassing and specific approach when the objective is to understand the broader context and associated environment The text highlights ecological assessment as the better answer because it is more specific and more precise when evaluating the environment Indirect assessment A such as interviews would provide subjective information and not a firsthand comprehensive account of the environment itself Functional analysis B focuses on identifying the function of specific problem behaviors for individuals and is not designed for a broad environmental understanding prior to intervention design

#89. When a Board Certified Behavior Analyst (BCBA) is developing a behavior-change program for a client, they are guided by ethical principles and best practices in the field. This includes considering the characteristics of various interventions. According to the foundational principles of Applied Behavior Analysis and ethical considerations for intervention selection, what approach should a BCBA prioritize when choosing and implementing intervention tactics?

This principle is often referred to as the least restrictive environment or the hierarchy of restrictiveness in intervention selection It mandates that behavior analysts should always start with interventions that are the least intrusive least restrictive and least costly both in terms of financial resources and potential side effects on the clients dignity and autonomy This approach is ethically grounded aiming to maximize client choice and minimize any potential adverse impacts More restrictive intrusive or costly interventions are only considered and implemented if data demonstrates that less restrictive alternatives have been thoroughly attempted and proven insufficient in achieving the desired behavioral outcomes This ensures a parsimonious and ethical approach to behaviorchange programming

#90. A Board Certified Behavior Analyst (BCBA) is planning to implement a Differential Reinforcement of Incompatible (DRI) or Alternative (DRA) behavior procedure to reduce a client’s problem behavior. Before implementing either DRI or DRA, what crucial factor must the BCBA ensure regarding the replacement behavior?

Differential Reinforcement of Incompatible DRI and Differential Reinforcement of Alternative DRA behaviors are powerful behaviorchange procedures because they teach and strengthen a replacement behavior while simultaneously decreasing the problem behavior A fundamental prerequisite for both DRI and DRA is that the replacement behavior must already be in the clients repertoire This means the client must be able to perform the desired alternative or incompatible behavior The goal of DRIDRA is to reinforce this existing behavior not to teach a new skill from scratch If the replacement behavior is not in the repertoire other teaching strategies like shaping modeling or prompting would be needed first to establish the skill Option A describes a common ethical consideration for implementing punishment procedures not differential reinforcement Option C describes Differential Reinforcement of Other DRO behavior where reinforcement is delivered when the problem behavior has not occurred for a specified period without necessarily requiring an alternative behavior Option D while functional communication training is a common application of DRA the ability to functionally communicate is not a universal prerequisite for all DRIDRA procedures any alternative behavior in the repertoire can be targeted

#91. Rachel, a Board Certified Behavior Analyst (BCBA), has been supervising Ross, a BCaBA, for seven months at Central Park ABA company. Over the course of their professional relationship, Rachel and Ross develop mutual romantic feelings and decide they wish to pursue a romantic relationship. According to the BACB’s Professional and Ethical Compliance Code for Behavior Analysts, what is the mandatory course of action Rachel and Ross must take if they decide to continue their romantic relationship?

The BACBs ethical guidelines strictly prohibit supervisors from engaging in romantic or intimate relationships with their supervisees This is to prevent conflicts of interest ensure professional boundaries and avoid any potential for exploitation or impairment of professional judgment If a romantic relationship develops the supervisor Rachel is ethically obligated to immediately terminate the supervisory relationship with the supervisee Ross and ensure that this termination is properly documented It is crucial that the supervisory duties cease entirely Unlike client relationships there is no mandated twoyear waiting period for relationships with former supervisees but the immediate cessation and documentation of the supervisory relationship are paramount Terminating Rosss employment is not a required ethical action only the supervisory role itself

#92. The true definition of imitation in Applied Behavior Analysis (ABA) requires a specific set of conditions for a behavior to be classified as such. Which of the following scenarios best exemplifies imitation according to its precise definition as described?

This question assesses a precise understanding of the definition of imitation which is often distinguished from more general modeling or observational learning in ABA According to the text the true definition of imitation requires two critical conditions 1 A behavior evoked by a nonverbal stimulus model The behavior must be prompted by observing anothers behavior a physical demonstration not by a verbal instruction or command 2 Occurs immediately after that stimulus The replicated behavior must follow the observed model without any significant delay Lets analyze each option against these criteria A Borah says High five to her friend who then immediately gives her a high five This is an example of a behavior evoked by a verbal discriminative stimulus SD High five not a nonverbal model While the response is immediate it fails the nonverbal stimulus criterion Therefore it is not considered imitation B Yesterday a student watched a video Today several hours later the student attempts a similar backflip While watching a video of a backflip could provide a nonverbal visual model the behavior occurs several hours later This significant delay violates the immediately after criterion for imitation This would be considered observational learning or delayed modeling but not imitation in the strict ABA sense C A child sees their older sibling stack three blocks to build a tower Immediately after the child picks up three blocks and stacks them in the same configuration This scenario perfectly aligns with the strict definition The older sibling stacking blocks serves as a nonverbal model a visual stimulus The childs subsequent stacking behavior is topographically similar looks like the model and occurs immediately after observing the model This is the clearest example of imitation D A client is instructed to clap your hands by their therapist and the client then claps their hands Similar to option A this behavior is evoked by a verbal SD clap your hands not a nonverbal model Thus it does not meet the criteria for imitation

#93. A behavior analyst is programming for the maintenance of interview skills with a client. The goal is to ensure the client can consistently apply these skills in various job interview scenarios over time, even after direct intervention decreases. Which of the following strategies would be least likely to effectively promote the long-term maintenance of these interview skills in naturalistic settings?

The question asks for the least likely effective strategy for maintenance Maintenance refers to the extent to which a learner continues to perform the target behavior after a portion or all of the intervention has been terminated Strategies that promote maintenance typically involve making the behavior resilient to changes in the environment and reducing reliance on the interventionist Option A Selfmonitoring and selfreinforcement These are selfmanagement strategies that empower the individual to manage their own behavior which is highly effective for maintenance Option B Varying conditions Introducing different interviewers settings and questions promotes generalization of the skills across various stimuli and contexts a critical component for longterm maintenance in natural environments Option C Fading prompts and intermittent reinforcement Fading prompts reduces the learners dependence on artificial supports and intermittent reinforcement schedules mimic the natural contingencies of reinforcement found in the real world making the behavior more resistant to extinction and promoting maintenance Option D Continuously conducting highly structured massedtrial practice with the same trainer in the clinic setting While massedtrial practice is excellent for initial skill acquisition continuing this highly structured specific and consistent format without variation or fading makes the learners behavior highly dependent on those specific training conditions This approach hinders maintenance because it does not prepare the client for the variability less frequent reinforcement and absence of the specific trainer that will be encountered in actual job interviews Therefore it is the least effective for promoting longterm maintenance in natural settings

#94. A BCBA has collaboratively developed a contingency contract for a 16-year-old client that involves the client, the client’s mother, and the BCBA as key parties. To maximize the contract’s effectiveness, ensure ongoing transparency, and facilitate consistent awareness of its terms for all involved, where would it be most appropriate to keep or display this contract?

A contingency contract is a written agreement between two or more parties that clearly specifies a contingency relationship between the completion of a specific behavior and access to a reinforcer For such a contract to be maximally effective all parties involvedthe client family members like the mother in this case and the behavior analystmust be continuously aware of its terms their respective responsibilities and the conditions for reinforcement Placing the contract in a prominent common area within the home such as on a refrigerator a family bulletin board or a shared living space ensures that it is easily visible and accessible to everyone This promotes transparency facilitates ongoing selfmonitoring and mutual accountability and provides a constant visual reminder of the agreedupon expectations and rewards This shared visibility significantly increases the likelihood of adherence and success for the entire contingency group Conversely keeping it solely with one party or in an inaccessible location diminishes its function as a transparent shared agreement making it less effective

#95. Jay, a newly certified BCBA, has been working with a client on independent dressing skills for several weeks. The client is now consistently dressing himself without assistance. However, Jay’s supervisor observes that the client’s improvement was likely due to the client watching YouTube tutorials on dressing and his parents providing intensive practice at home, rather than Jay’s specific intervention plan. Given this information, what critical aspect does Jay’s intervention lack, and why is this problematic in an ABA context?

Internal validity is a fundamental concept in experimental design within Applied Behavior Analysis ABA referring to the extent to which an experiment demonstrates that changes in the dependent variable the clients behavior are directly attributable to the independent variable the intervention and not to extraneous variables In this scenario the supervisors observation suggests that external factors YouTube tutorials parental practice are more likely responsible for the clients improved dressing skills rather than Jays specific intervention This means that Jays intervention lacks internal validity because experimental control was not established other variables confounded the outcome making it impossible to confidently conclude that the intervention caused the behavior change Without internal validity the effectiveness of the intervention cannot be credibly determined External validity in contrast refers to the generalizability of findings to other settings people or behaviors Social validity pertains to the social significance of the target behavior the appropriateness of the procedures and the social importance of the results Observer validity or interobserver agreement relates to the consistency and accuracy of data collection by multiple observers None of these align with the problem of not being able to attribute the behavior change to the specific intervention

#96. A BCBA is implementing a changing criterion design to systematically increase the number of independent work tasks a client completes each hour. The criterion is initially set at 2 tasks per hour and is then sequentially increased to 4, 6, and finally 8 tasks per hour. To demonstrate experimental control within this design, what must be observed regarding the client’s behavior?

In a changing criterion design experimental control is demonstrated when the clients behavior closely matches or tracks the changing criterion levels This means that as the criterion for the target behavior eg number of tasks completed is systematically increased or decreased the clients behavior also systematically increases or decreases in direct correspondence with those criterion changes This concurrent and systematic relationship between the criterion and the behavior provides strong evidence that the intervention the criterion change is responsible for the observed changes in behavior thereby demonstrating experimental control If the behavior deviates significantly from the criterion eg consistently drastically above or below or shows no systematic pattern experimental control would not be established

#97. In a scenario where Lucy consistently pulls a football away as Charlie attempts to kick it, Charlie expresses his frustration, stating that Lucy ‘always’ does this. Despite Lucy’s action occurring immediately after Charlie’s attempt to kick, his future behavior of attempting to kick the ball does not decrease nor increase he continues to try. In this specific context, Lucy’s action of pulling the football away functions as what?

A consequence is defined as any stimulus change that occurs after a behavior For a consequence to be functionally classified as a reinforcer or a punisher it must produce a measurable change in the future frequency or probability of the behavior it follows Reinforcement increases the future frequency of behavior while punishment decreases it The text explicitly states that the behavior hasnt changed at all so its not punishing him but its not reinforcing him either Therefore Lucys action while a consequence it follows the behavior is not functionally a reinforcer or a punisher It is best described as a neutral consequence because it does not alter Charlies future kicking behavior Option A is incorrect because immediate presentation alone does not define reinforcement an increase in future behavior frequency is required Option B is incorrect because Charlies verbal expression of frustration and his past experiences while providing context do not define the functional effect of the current consequence on the future rate of his kicking behavior the text clarifies that the behavior did not decrease Option C is incorrect because a discriminative stimulus SD is an antecedent stimulus that occasions a response not a consequence that follows it

#98. A family is at a restaurant, and initially, a young adult named Alex wanted to order the fish. However, when Alex’s brother commented that the steak looked particularly good, Alex’s preference shifted, and upon the waiter’s arrival, Alex ordered the steak. In this scenario, what concept best describes the brother’s comment about the steak?

A Motivating Operation MO is an environmental variable that alters the effectiveness of a stimulus as a reinforcer valuealtering effect and alters the current frequency of all behavior that has been reinforced by that stimulus behavioraltering effect In this scenario Alexs brothers comment the steak looked good functioned as an MO It increased the value of the steak as a reinforcer for Alex valuealtering effect and consequently evoked the behavior of ordering the steak behavioraltering effect It is not a Discriminative Stimulus SD because an SD signals the availability of reinforcement for a particular behavior but does not alter the value of the reinforcer itself eg the waiter asking What would you like to order would be an SD for ordering as it signals that ordering will be reinforced by receiving food A Conditioned Stimulus CS is relevant to classical conditioning where a previously neutral stimulus acquires the ability to elicit a response through association A reinforcer is the consequence that follows a behavior and increases its future probability not the antecedent that alters its value

#99. Jim repeatedly rings a small bell immediately before offering Dwight an Altoid, which causes Dwight’s mouth to dry up. After two weeks of this pairing, Dwight’s mouth begins to dry up merely at the sound of the bell, even without the Altoid. In this classic respondent conditioning example, what serves as the conditioned stimulus (CS)?

This scenario perfectly illustrates respondent conditioning Initially the small bell is a neutral stimulus NS that does not evoke the mouthdrying response The Altoid however is an unconditioned stimulus US because it naturally and automatically causes Dwights mouth to dry up unconditioned reflex UR Through repeated pairing the previously neutral small bell becomes associated with the Altoid Consequently the small bell acquires the ability to evoke the mouthdrying response on its own At this point the small bell transitions from being an NS to a conditioned stimulus CS and Dwights mouth drying up specifically in response to the bell becomes the conditioned reflex CR Option A the Altoid is the unconditioned stimulus Option B Dwights mouth drying up when he hears the bell is the conditioned reflex Option D Dwights desk is irrelevant to the conditioning process described

#100. A Board Certified Behavior Analyst (BCBA) has implemented a multi-component intervention package to address a client’s severe problem behavior. The intervention has been effective in reducing the target behavior, but the BCBA now wants to systematically determine which specific elements or combinations of elements within the package are most responsible for the observed behavior change. Which of the following analytical procedures would be most appropriate for the BCBA to conduct?

A component analysis is an experimental design strategy used to evaluate the relative contributions of individual components within a multicomponent intervention package The goal is to identify which parts of the intervention are necessary sufficient or most effective in producing the desired behavior change In this scenario the BCBA wants to know what intervention is responsible whats working what isnt which perfectly aligns with the purpose of a component analysis A treatment integrity check A assesses whether the intervention is being implemented as planned not which components are effective A baseline assessment C is conducted prior to intervention to establish a preintervention level of behavior and would not help evaluate components of an already implemented successful intervention A functional analysis D is used to identify the function eg attention escape tangibles automatic reinforcement of a problem behavior not to dissect the effectiveness of intervention components

#101. Ben is observing a written prompt, ‘Please read page 10,’ and subsequently vocalizing the words ‘Please read page 10.’ The speaker in the provided text identifies this behavior as a specific verbal operant. Based on this description and the principles of Verbal Behavior, what type of verbal operant is Ben engaging in?

A textual operant is a verbal operant that is evoked by a nonauditory verbal stimulus eg written words symbols and has pointtopoint correspondence between the stimulus and the response but lacks formal similarity In this scenario Ben is reading written words Please read page 10 and vocalizing them This demonstrates control by a nonauditory verbal stimulus with pointtopoint correspondence each word written corresponds to a word spoken An intraverbal A is a verbal operant evoked by a verbal stimulus with no pointtopoint correspondence and no formal similarity eg answering a question An autoclitic B is a secondary verbal operant that modifies the impact of a primary verbal operant often by commenting on it or indicating its strength An echoic D is a verbal operant evoked by an auditory verbal stimulus with pointtopoint correspondence and formal similarity eg repeating what someone else said

#102. A newly certified behavior analyst is reviewing the ethical guidelines regarding professional conduct. They encounter a scenario where a client’s parent, grateful for the progress their child has made, offers the behavior analyst a small token of appreciation. Based on current ethical guidelines, which of the following statements is true regarding a behavior analyst accepting gifts from a client or supervisee?

This question addresses the ethical guidelines surrounding gifts in ABA practice specifically from clients or supervisees The BACB Professional and Ethical Compliance Code for Behavior Analysts specifically section 106 Multiple Relationships and Conflicts of Interest provides guidance The code clarifies that behavior analysts are generally allowed to give and accept gifts of minimal monetary value eg less than 10 and not on a frequent or ongoing basis The core principle is to avoid creating multiple relationships or conflicts of interest that could impair the behavior analysts objectivity or exploit the clientsupervisee Accepting gifts of substantial value or frequency can blur professional boundaries create a sense of obligation or be perceived as a conflict of interest potentially compromising the professional relationship and the quality of services Therefore small infrequent gifts of minimal monetary value are generally permissible as they are less likely to pose an ethical dilemma Its crucial for behavior analysts to always err on the side of caution and prioritize the clients best interest and the integrity of the professional relationship ensuring that the gift does not create a conflict of interest or compromise their professional judgment

#103. You consistently make tea every morning for your family, and everyone drinks a cup. In the afternoon, you often consider making a second pot of tea. On three separate occasions, you ask your family if they would like any afternoon tea, and each time they unequivocally tell you ‘no.’ After the third instance of asking and receiving a ‘no,’ you stop asking them in the afternoon and instead just make a pot for yourself. What behavioral principle most accurately explains the change in your future behavior of asking your family if they want afternoon tea?

Your behavior of asking your family if they want tea in the afternoon decreased you stopped asking after they repeatedly said no A decrease in the future probability of a behavior indicates a punishment procedure has occurred We must then determine if its positive or negative Since the no response from your family was added to the environment it appeared immediately after your behavior and resulted in a decrease in your future behavior of asking it represents a positive punishment procedure Extinction A involves the withholding of reinforcement for a behavior that was previously reinforced in this scenario your afternoon asking behavior was never reinforced by family members accepting tea so there was no reinforcement to withhold Negative punishment C would involve the removal of a stimulus following a behavior which did not occur here Positive reinforcement D would result in an increase in the future probability of the behavior which is the opposite of what happened

#104. Johnny’s favorite breakfast is pancakes. When Johnny is served pancakes, he consistently drowns them in maple syrup, using an excessive amount. In response to this behavior, Johnny’s mom removes the maple syrup from the table. Based solely on the information provided, what can be definitively stated about Johnny’s mom’s intervention?

This question highlights a fundamental principle in Applied Behavior Analysis the definition of punishment and reinforcement is entirely functional meaning it is determined solely by its observed effect on future behavior While the removal of maple syrup after Johnnys behavior of drowning pancakes in syrup looks like negative punishment Option A due to the removal of a desired item we cannot definitively classify it as such without observing Johnnys future behavior If Johnnys behavior of using excessive syrup decreases in the future then the intervention was indeed punishment However if Johnnys behavior increases or stays the same then the intervention was not punishment regardless of the moms intention Similarly we cannot determine if its negative reinforcement for the mom Option B without observing changes in her future behavior ie whether she is more likely to remove syrup in the future Therefore the only thing that can be definitively stated from the given information is that the removal of syrup is a consequence Option C that immediately followed Johnnys behavior The ultimate effect on Johnnys future behavior remains unknown from the provided text rendering a determination of punishment or reinforcement impossible

#105. After completing a thorough behavioral assessment for a new client, a BCBA is tasked with developing a comprehensive and ethically sound behavior-change intervention plan. According to best practices in Applied Behavior Analysis and the professional ethical code, which of the following crucial factors must inform the design and implementation of these interventions?

Developing effective and ethical behaviorchange interventions requires a multifaceted approach The text highlights three indispensable factors 1 Client preference This emphasizes the ethical principle of client dignity and the importance of involving the client and relevant stakeholders in treatment decisions to the fullest extent possible Interventions are more likely to be successful and maintainable when they align with the clients values and preferences 2 Assessment results Behavioral assessments eg functional analyses preference assessments skill assessments provide the empirical data necessary to understand the target behavior its function and the clients current skill repertoire Interventions must be directly derived from these results to be effective 3 Scientific evidence ABA is an evidencebased practice meaning interventions must be grounded in empirically supported research and proven methods This ensures that the chosen strategies have demonstrated effectiveness and adhere to the principles of behavior science Basing interventions on only one or two of these factors or on convenience would lead to incomplete or potentially unethical treatment plans

#106. In the scientific pursuit of Applied Behavior Analysis, what is the primary goal of the description level of understanding behavior?

The scientific goals of ABA mirroring those of science in general are typically categorized into three progressively deeper levels description prediction and control Description C This is the foundational level involving systematic observation measurement and documentation of behavior and the environmental events that surround it The goal is to collect objective facts about what happened when it happened and under what conditions without interpreting why it happened or attempting to manipulate variables This forms the empirical basis for further scientific inquiry Prediction B This level involves identifying correlations between events Based on descriptive data we can predict that if one event occurs another event is likely to follow but it does not establish a causeandeffect relationship Control A D This is the highest level of scientific understanding achieved when a functional relationship is demonstrated It involves manipulating an independent variable environmental variable to reliably produce a change in a dependent variable behavior Identifying functional relationships A is the outcome of achieving control and manipulating variables to influence behavior change D is the process of achieving it Therefore collecting objective observable data is the hallmark of the descriptive goal

#107. A Board Certified Behavior Analyst (BCBA) has amassed nearly a decade of experience, primarily focusing on interventions for individuals diagnosed with Autism Spectrum Disorder. Recently, the BCBA was contacted to consult for a new client with a severe intellectual disability, a population with whom the BCBA has very limited direct experience or specialized training. If the BCBA decides to accept this consultation, what is the first and most critical ethical step they must undertake to ensure competent service delivery, in accordance with professional ethical guidelines?

The ethical code for behavior analysts mandates that practitioners only provide services within their areas of competence When presented with a case outside their primary or established area of expertise eg transitioning from extensive experience with autism to a client with a severe intellectual disability without prior experience the first and most critical ethical step if choosing to accept the case is to obtain appropriate training and supervision from a professional competent in that specific area This ensures that the BCBA develops the necessary skills and knowledge to effectively and ethically serve the new client population While conducting assessments A and C are crucial steps in the intervention process they cannot ethically precede the BCBA ensuring their own competence to work with the client Declining the case D is an acceptable option if competence cannot be attained but it is not the first required step if the BCBA intends to take the case and acquire the necessary competency

#108. A Board Certified Behavior Analyst (BCBA) is reviewing the data for a client’s target behavior. The data consistently shows no significant change or decrease in the problem behavior over the past few weeks, suggesting the intervention is not effective. However, the client’s parent expresses that they’ve noticed a considerable improvement in the behavior at home and are very pleased with the progress. Given this discrepancy between subjective report and objective data, what is the BCBA’s most appropriate initial step according to best practice in applied behavior analysis?

When there is a discrepancy between collected data showing no change and a stakeholders perception reporting improvement the first and most appropriate step for a BCBA is to investigate the fidelity of treatment implementation and the accuracy of data collection As the text states if the clients data is saying I dont see a change and the data is telling you something drastically different you need to go ahead and check for treatment Fidelity you have to make sure your technicians are taking data correctly This involves directly observing the technicians in action not just asking them to ensure they are applying the intervention correctly measuring the target behavior accurately and that the data is reliable and valid Ignoring the data Option A or blindly trusting the parents report Option C would be unethical and contradict datadriven decisionmaking Simply explaining data to the parent Option B does not address the fundamental issue of data validity or treatment integrity when theres a clear discrepancy

#109. Holly works as a receptionist at a law firm. Each morning, she is responsible for sending her boss his daily schedule. She has consistently performed this task by texting the schedule to his mobile phone for months. One morning, due to a technical issue with her phone, Holly is unable to text the schedule. Instead, she successfully emails the schedule to her boss, fulfilling the same function. What does Holly’s ability to adapt her behavior in this situation best illustrate, and what are its key characteristics?

Response generalization occurs when a learner emits varied forms of behavior that all produce the same effect or outcome in the presence of the same stimulus or context In this scenario the stimulus condition remains constant Holly needs to send her bosss daily schedule Her initial learned response was texting the schedule However when that specific response was unavailable she effectively demonstrated a different topographically distinct response emailing the schedule that served the identical function of delivering the schedule to her boss This adaptation highlights response generalization the behaviors form is flexible but its function remains consistent under the same stimulus control Stimulus generalization conversely involves performing the same response in the presence of different but similar stimuli Maintenance refers to the continued performance of a learned behavior over time after an intervention has been withdrawn Stimulus equivalence describes the emergence of accurate responding to untrained and unreinforced stimulusstimulus relationships eg reflexivity symmetry transitivity which is not depicted here

#110. During a multidisciplinary team meeting, a BCBA is collaborating with other professionals and stakeholders to determine appropriate intervention strategies for programming generalization of a newly acquired skill. Considering best practice and ethical guidelines for intervention selection, how should the team prioritize and approach the selection of these interventions?

When selecting interventions particularly for complex processes like generalization ethical and practical considerations dictate starting with the least intrusive and least costly options This approach aligns with the principle of least restrictive environment which emphasizes interventions that promote independence and integrate the client into natural settings with minimal artificial support Highly intrusive tactics such as heavy prompting can inadvertently hinder generalization by making the behavior dependent on artificial cues thus requiring extensive and often timeconsuming fading procedures Furthermore the text explicitly highlights that cost and available resources are critical factors that must always be considered especially when working with diverse client populations or limited funding A responsible and effective approach involves a systematic hierarchy of interventions beginning with the least restrictive least intrusive and most naturalistic methods and only escalating to more intensive options when the initial less intrusive strategies prove insufficient

#111. A researcher is conducting a study in a highly controlled laboratory setting, systematically manipulating specific environmental variables to understand fundamental principles governing how organisms learn and adapt their behaviors. The primary goal of this research is to advance basic scientific knowledge about behavior, without immediate consideration for its practical application to complex human social problems. This type of basic research, often associated with B.F. Skinner’s initial scientific endeavors, is characteristic of which domain within behavior analysis?

The text outlines four interconnected domains of behavior analysis Philosophy of Behaviorism Experimental Analysis of Behavior EAB Applied Behavior Analysis ABA and Professional Practice of Behavior Analysis Experimental Analysis of Behavior EAB is specifically defined as basic research conducted in highly controlled environments typically laboratories to identify and describe the fundamental principles of behavior It focuses on discovering basic behavioral processes and relationships rather than immediate application to socially significant problems Applied Behavior Analysis ABA Option A involves applying behavioral principles to improve socially significant behaviors in realworld natural settings The Professional Practice of Behavior Analysis Option B refers to the direct implementation of ABAderived treatments and interventions by practitioners The Philosophy of Behaviorism Option C addresses the theoretical conceptual and philosophical underpinnings of behavior science including its assumptions worldview and implications Given the description of basic research in a controlled laboratory environment with a focus on fundamental principles EAB is the correct domain

#112. Albert, a 7-year-old boy, consistently demonstrates a high preference for helping his teacher with tasks such as organizing supplies during recess. However, Albert struggles with initiating interactions with his peers and making friends, a behavior his parents wish to increase. To leverage Albert’s high-preference activity to strengthen his peer interaction skills, a behavior analyst plans to implement the Premack Principle. Which of the following strategies best exemplifies the appropriate application of the Premack Principle in this scenario?

The Premack Principle often referred to as Grandmas Rule states that a highprobability behavior a preferred activity can be used as a reinforcer for a lowprobability behavior a less preferred activity It involves making access to the preferred activity contingent upon the completion of the less preferred activity In this case helping the teacher is Alberts highprobability behavior and initiating peer interactions is his lowprobability behavior Option B correctly applies the Premack Principle by making access to the preferred activity helping the teacher contingent on the performance of the less preferred activity social initiations with peers Option A is a form of general positive reinforcement but it doesnt utilize the contingency inherent in the Premack Principle Option C describes a token economy which is a different behaviorchange system although it can incorporate preferred activities Option D is an aversive approach that involves blocking access without a clear contingency for the desired behavior and it does not represent the Premack Principle it could even be considered a form of punishment or deprivation which may have negative side effects

#113. A behavior analyst is discussing the foundational principles of behaviorism with a new supervisee. They are specifically addressing how different schools of thought within behaviorism regard ‘private events’ (e.g., thinking, feeling). Which statement accurately distinguishes radical behaviorism from methodological behaviorism concerning private events?

Radical behaviorism as established by BF Skinner is the philosophy of behavior science that we practice in ABA A core tenet of radical behaviorism is the acknowledgment that private events eg thoughts feelings sensations are indeed a form of behavior subject to the same laws of learning as publicly observable behaviors While we may not directly observe or measure them in the same way as overt behaviors we do not dismiss their existence or their behavioral nature Methodological behaviorism which preceded radical behaviorism took a more stringent view focusing solely on publicly observable behaviors and largely ignoring or denying the scientific relevance of private events The text explicitly states radical behaviorism acknowledges private events as behavior methodological behaviorism which ignored private offensive behavior they didnt acknowledge private events as behavior

#114. A BCBA is conducting a Multiple Stimulus Preference Assessment with Replacement (MSW) for a client named Alex to identify preferred items. In the first array, Alex is presented with a toy truck (T), play-doh (P), a coloring book (C), bubbles (B), and a tablet (A). Alex selects the toy truck (T). Which of the following describes the correct procedure for the next trial in this MSW assessment?

A Multiple Stimulus Preference Assessment with Replacement MSW is a method used to identify a hierarchy of preferred items by allowing a learner to choose from an array of three or more stimuli The key differentiating feature of an MSW as explained in the text is what happens after an item is chosen 1 Chosen Item The item that the learner selects is put back into the array for the next trial This ensures that a highly preferred item can be chosen repeatedly across trials 2 Unchosen Items The items that were not chosen in the current trial are replaced with new items in the subsequent trial This allows for a continuous sampling of novel stimuli to maintain engagement and provide a broader assessment of preferences Therefore in the given scenario since Alex chose the toy truck T it must be returned to the array The unchosen items playdoh coloring book bubbles tablet are then swapped out for completely new different items The correct answer accurately reflects this procedure Lets look at why the other options are incorrect Option 1 describes a Multiple Stimulus Preference Assessment Without Replacement MSWO where the chosen item is removed from the array Option 3 incorrectly states that the unchosen items are presented again In an MSW the unchosen items are replaced with new ones Option 4 is a nonstandard procedure that does not align with either MSW or MSWO methodologies as it suggests keeping the chosen item in the array but making it unavailable which would confound the assessment

#115. A BCBA is considering accepting a gift from a supervisee who recently passed their certification exam, expressing gratitude for the BCBA’s guidance. The supervisee offers a new pair of designer jeans worth approximately 150. According to the ethical guidelines for behavior analysts, specifically regarding gifts, what is the appropriate course of action for the BCBA?

The BACBs Professional and Ethical Compliance Code for Behavior Analysts explicitly states that the same rules for accepting gifts from clients and stakeholders also apply to supervisors and superviseestrainees This is a critical measure to prevent the formation of dual relationships which can impair objectivity and to avoid conflicts of interest The ethical guidelines stipulate that gifts if accepted must be of nominal value generally understood to be less than 10 and not be offered or received on an annual or routine basis A pair of designer jeans valued at 150 far exceeds this nominal value making its acceptance unethical While supervisees are not prohibited from giving gifts they must adhere to these same strict guidelines regarding value and frequency

#116. A Board Certified Behavior Analyst (BCBA) is planning a research study using a multiple baseline design to evaluate the effectiveness of a new social skills curriculum. Based on the fundamental principles and typical applications of this experimental design, which of the following is NOT a recommended application for evaluation using a multiple baseline design?

A multiple baseline design is designed to demonstrate experimental control by introducing a single independent variable intervention sequentially across two or more baselines settings participants or behaviors The core strength of this design is its ability to show that the introduction of that specific intervention leads to a change in the dependent variable in a staggered fashion Therefore it is not recommended for evaluating multiple distinct interventions simultaneously within the same design as this would violate the principle of having a single independent variable and make it difficult to determine which specific intervention was responsible for observed changes Options A C and D are all standard and appropriate applications of a multiple baseline design

#117. Tiffany, a dedicated RBT, consistently implements her client’s social skills treatment plan with high fidelity, actively seeks and integrates feedback from her supervisor, and makes necessary adjustments to her service delivery. Despite six months of consistent intervention, the client still struggles significantly to form friendships and initiate or maintain conversations. Given this outcome, which dimension of applied behavior analysis is notably lacking or insufficient in the service being provided?

The scenario describes an intervention that is being implemented correctly and consistently high fidelity takes feedback adjusts service However the critical issue is that the intervention is not producing the desired social skills changes in the client client still does not have friends and is unable to hold a conversation This directly points to a lack of effectiveness The Effective dimension of ABA as described by Baer Wolf and Risley 1968 requires that interventions produce significant practical and meaningful changes in behavior While the plan may be targeting observable behaviors Behavioral be based on ABA principles Conceptually Systematic and be clearly written enough for consistent implementation Technological its ultimate failure to achieve the desired outcomes indicates a deficiency in its effectiveness

#118. Which specific branch of behavior analysis is primarily focused on developing and implementing technologies for behavior change that address behaviors with direct and immediate importance to the individual and society, commonly referred to as ‘socially significant behaviors’?

The field of behavior analysis is often understood through its three main branches Experimental Analysis of Behavior EAB Applied Behavior Analysis ABA and Conceptual Analysis of Behavior CAB The text specifies that the branch which develops technologies of behavior change that are related to socially significant behaviors is the focus This precisely defines Applied Behavior Analysis ABA EAB is concerned with basic research to discover fundamental principles of behavior in controlled laboratory settings CAB focuses on the philosophical theoretical and historical underpinnings of behaviorism and its concepts Radical Behaviorism is the philosophy of science that underpins all branches of behavior analysis particularly BF Skinners approach rather than a branch focused on technology development for socially significant behaviors itself

#119. A Board Certified Behavior Analyst (BCBA) is working with an adolescent client to develop essential interview skills, including greeting, answering questions clearly, and maintaining eye contact. After the client demonstrates proficiency in a controlled teaching environment, the BCBA aims to program for the maintenance of these skills in the natural environment, ensuring they persist once direct teaching stops. Which of the following strategies would be least likely to be effective in promoting long-term maintenance of the interview skills?

Maintenance refers to the extent to which a learner continues to perform the target behavior after a portion or all of the intervention has been terminated To promote maintenance the goal is to shift the control of the behavior from contrived reinforcement to natural reinforcement Using a continuous reinforcement CRF schedule for an extended period is least likely to be effective for maintenance because it makes the behavior highly sensitive to the removal of reinforcement Once the CRF schedule is discontinued the behavior is prone to rapid extinction as the learner has not experienced the intermittent and often delayed reinforcement that characterizes natural environments Effective strategies for maintenance include fading contrived reinforcement to intermittent schedules arranging for natural reinforcement programming for generalization eg across people settings stimuli and conducting occasional maintenance checks with varied conditions The other options A B D are all effective strategies for promoting maintenance and generalization because they involve contact with natural reinforcement varied environmentspeople and intermittent schedules of reinforcement implied by fading teaching and using occasional checks

#120. Elvis, a talented musician, has not practiced playing his guitar or taken lessons for several months. Despite this, he occasionally picks up his guitar to see if he can still play his favorite songs. He is not performing for an audience or attempting to play in new locations he is simply checking his own ability. In the context of Applied Behavior Analysis, what concept is Elvis essentially testing?

Maintenance refers to the extent to which a learner continues to perform a target behavior after a portion or all of the intervention responsible for the behaviors initial appearance andor increase has been terminated In this scenario Elvis is assessing if his guitarplaying skills have persisted over time even though he is no longer actively practicing or receiving instruction This is a classic example of testing for maintenance Stimulus generalization would involve performing the skill in different settings or with different stimuli eg different guitars different venues which Elvis is explicitly not doing Response generalization would involve performing new untaught responses that are functionally equivalent to the trained response Behavior contrast describes a phenomenon where a change in a schedule of reinforcement in one setting leads to an opposite change in the rate of behavior in another setting Neither of these accurately describes Elviss action of checking if a previously learned skill still exists after a period of no practice

#121. Antonio, a piano student, has successfully learned to play the correct sequence of notes for a complex Mozart piece. His piano teacher’s next goal is to refine Antonio’s performance by teaching him to play specific notes with varying magnitudes (e.g., certain notes louder, others softer) to achieve a more expressive sound. Which type of shaping procedure is the piano teacher most likely utilizing to achieve this new goal?

Shaping involves differentially reinforcing successive approximations of a target behavior In this scenario Antonio has already mastered the form or topography of playing the correct notes The teachers goal is not to change what Antonio plays but how he plays it specifically the magnitude loudnesssoftness of the notes Shaping within response topographies is used when the basic form of the behavior is already present but a specific dimension of that behavior such as duration latency force or magnitude needs to be modified or refined Shaping across response topographies on the other hand would involve reinforcing entirely new forms of behavior progressively moving towards a desired novel response Since the topography playing the correct notes is already established and only a dimension magnitude is being altered this is a clear example of shaping within response topographies

#122. A BCBA is working with a client on a behavior contract designed to increase academic task completion. The client’s brother, who is not a client, begins to consistently sabotage the client’s attempts to successfully complete the contract. Which of the following is the best next step for the BCBA to take in this scenario?

This scenario highlights the importance of working within ones scope of practice and addressing environmental variables that affect the clients behavior The BCBAs primary responsibility is to their client The brother is not the BCBAs client so providing direct treatment recommendations for his behavior or collecting data with the intent of directly treating him would be outside the BCBAs professional and ethical scope Ignoring the environmental disruption caused by the sibling is not appropriate as it directly impedes the clients progress and the effectiveness of the intervention Similarly rewarding the client for not completing the contract due to external factors would undermine the integrity and purpose of the behavior contract itself The most appropriate and ethical course of action is to collaborate with the clients parents stakeholders to discuss the siblings behavior and its impact The BCBA can then offer recommendations for environmental modifications or strategies that the parents can implement to mitigate the siblings disruptive behavior thereby supporting the clients ability to successfully meet the terms of their behavior contract This approach focuses on adjusting the clients environment to promote their success while adhering to ethical guidelines regarding client boundaries

#123. A behavior analyst is observing a client’s responding patterns on a schedule of reinforcement. The client engages in a high, steady rate of responses, then pauses immediately following reinforcement, and subsequently resumes the high rate of responding. This pattern is characteristic of which schedule of reinforcement?

The described pattern of respondinga high steady rate followed by a postreinforcement pauseis a hallmark characteristic of a Fixed Ratio FR schedule of reinforcement On an FR schedule reinforcement is delivered after a fixed number of responses Learners often complete the required responses quickly receive reinforcement then take a brief break postreinforcement pause before beginning the next ratio requirement This pattern is sometimes referred to as a scallop effect if viewed over a longer period though the text emphasizes the high steady rates with the post reinforcement pause Variable Interval VI schedules produce a moderate steady rate of responding with little or no postreinforcement pause Variable Ratio VR schedules produce high steady rates of responding with no postreinforcement pause as the uncertainty of when reinforcement will occur encourages continuous responding Fixed Interval FI schedules typically produce a scalloped pattern where responding increases as the end of the interval approaches followed by a pause after reinforcement but the rate is not a high steady rate until the end of the interval

#124. A BCBA is working to reduce a client’s repetitive requests for snacks throughout the day. During the initial baseline data collection phase, the behavior analyst observes a consistent and clear decreasing trend in the client’s snack requests. Considering this desired trend is occurring before any intervention has been implemented, what is the most appropriate next step for the BCBA?

When baseline data for a target behavior shows a trend in the desired direction eg a decreasing trend for a behavior targeted for reduction it is generally most appropriate to continue collecting baseline data The rationale is that the desired change is already occurring potentially due to unknown or uncontrolled variables Intervening prematurely might obscure the true functional relationship if the behavior change is not a result of the planned intervention By continuing baseline the BCBA can determine if the trend is stable sustainable or merely an anomaly Intervention should be considered if the desired trend reverses eg snack requests begin to increase again or if the behavior stabilizes at an unacceptable level indicating that the natural environmental factors are no longer sufficient to maintain the desired change Stopping data collection or shifting to another behavior prematurely would be unethical and not datadriven as it would prevent the analyst from confirming the stability and generality of the observed change

#125. A Board Certified Behavior Analyst (BCBA) is designing an intervention to increase a client’s sustained on-task behavior during independent academic work. The goal is for the client to remain engaged for the entire duration of pre-defined intervals. To accurately measure progress toward this goal and ensure the data reflects consistent engagement, which discontinuous measurement procedure would be most appropriate to select?

When the goal is to increase the duration or sustained presence of a behavior such as ontask behavior for the entire interval Whole Interval Recording is the most appropriate discontinuous measurement procedure In Whole Interval Recording the behavior is only recorded as occurring if it is present throughout the entire predetermined interval This method tends to underestimate the occurrence of behavior making it sensitive to increases in duration and thus ideal for behaviors targeted for increase Momentary Time Sampling only records behavior at the very end of an interval which can lead to overestimation or underestimation and may not provide a consistent picture of sustained behavior Partial Interval Recording records the behavior if it occurs at any point during the interval even for a single second This method tends to overestimate the occurrence of behavior which would be misleading if the goal is sustained engagement as a brief instance of ontask behavior would count the same as sustained ontask behavior Event recording or frequency recording is used for behaviors that have a clear beginning and end and occur discretely which is not suitable for measuring sustained ontask behavior across an interval

#126. Mr. Griffin manages a toy factory with 50 employees. To monitor employee engagement, he walks through the production area every hour on the hour and observes who is and isn’t engaged in their assigned duty, recording his observations on a clipboard. Considering the large group size and the specific timing of his observations, what type of measurement procedure is Mr. Griffin utilizing?

Mr Griffin is utilizing a Planned Activity Check PLACHECK PLACHECK is a discontinuous measurement procedure specifically designed for observing and measuring behavior in groups It involves observing a group of individuals at specific predetermined times eg every hour on the hour At the moment of observation the observer quickly scans the group and notes how many individuals are engaged in the target behavior This method is practical and efficient for large groups where continuous observation of each individual would be infeasible for a single observer While it shares some conceptual similarities with whole interval recording in that behavior is observed at a specific point or for the duration of a brief interval PLACHECK is tailored for group settings Partial interval recording requires observing for the entire interval and marking if the behavior occurs at any point during it which would be difficult for 50 employees simultaneously Whole interval recording requires the behavior to occur throughout the entire interval Permanent product is incorrect because Mr Griffin is directly observing behavior in realtime not the lasting effects or tangible outcomes of the behavior

#127. A BCBA is initiating services for a 5-year-old client who frequently makes requests for snacks throughout the day, often outside of scheduled meal times. The BCBA’s identified primary target behavior is to reduce the overall number of snack requests. During the initial baseline data collection phase, before any intervention has been implemented, the collected data consistently shows a clear and stable decreasing trend in snack requests, moving steadily towards the desired reduction goal. Given this observation, what is the most appropriate next step for the BCBA?

When baseline data for a target behavior that is meant to be reduced shows a decreasing trend and this trend is moving in the desired direction the most appropriate and empirically sound next step is to continue collecting baseline data The purpose of baseline is to establish a stable and accurate representation of the behaviors occurrence prior to intervention If the behavior is already improving intervening prematurely could mask the natural variability or an unobserved change in the environment that is causing the decrease Its essential to determine if the desired change can be maintained or achieved without formal intervention If the behavior continues to decrease and reaches the desired goal during baseline an intervention might be unnecessary or a modified less intensive approach might be considered Option A is incorrect because intervening immediately might lead to a false assumption that the intervention caused the decrease or it might disrupt a naturally occurring improvement Option C is premature the behavior may still require monitoring or a maintenance plan and the improvement might not be stable Option D is inappropriate and unethical as punishment procedures should be considered only after less intrusive methods have been exhausted and a significant clinical need is established neither of which is indicated when a behavior is already decreasing

#128. While on a challenging solo hike, Joanne finds herself approaching a steep incline towards the summit. To maintain her motivation and physical effort, she repeatedly mutters to herself, ‘Just keep climbing, just keep climbing, you’re almost there.’ This internal, automatic verbal behavior that serves to prompt and sustain her efforts is an example of what behavioral strategy?

Selfinstruction is a type of selfmanagement strategy where an individual provides verbal prompts or cues to themselves to guide encourage or evoke a specific behavior It is an automatic process where ones own verbal behavior functions as an antecedent to influence their subsequent actions often to initiate or maintain a desired response Joannes verbalizations are not directed at another person rather they are selfdirected and designed to prompt and sustain her own climbing behavior An intraverbal is a verbal operant where the response is evoked by a verbal stimulus and there is no pointtopoint correspondence between the stimulus and the response eg answering a question or filling in the blank It typically involves social mediation Listener behavior refers to nonverbal behavior that occurs in response to the verbal behavior of others eg following instructions complying with requests It is also socially mediated Selfmonitoring involves systematically observing and recording ones own behavior or progress towards a goal often with the intent to change the behavior which is different from simply delivering a verbal prompt to oneself to act

#129. A young child learns to identify and verbally label both a photograph of a butterfly and a drawing of a hot air balloon by saying, ‘It flies’ Despite the lack of obvious physical or topographical similarities between a butterfly and a hot air balloon, these two distinct visual stimuli consistently evoke the same verbal response. This phenomenon is an example of the formation of what type of stimulus class?

An arbitrary stimulus class is a set of stimuli that evoke the same response but do not share common physical features or topographies The members of the class are grouped together based on their shared function or the common responses they evoke rather than on their physical form In this scenario a butterfly and a hot air balloon are topographically dissimilar they look very different yet both consistently function to evoke the verbal response It flies This grouping based on a shared function flying despite visual differences is a defining characteristic of an arbitrary stimulus class A response class refers to a group of responses that produce the same effect on the environment or serve the same function eg different ways to open a door The question is specifically asking about the classification of stimuli not responses A feature stimulus class sometimes referred to as a concept in ABA consists of stimuli that share common physical features or properties eg all red objects all round objects all objects with wings Butterflies and hot air balloons do not share significant topographical features that would place them in a feature stimulus class based on their appearance alone A consequence class is not a standard or formal term in Applied Behavior Analysis consequences are events that follow behavior and affect its future probability whereas these are antecedent stimuli

#130. A driver is traveling significantly above the posted speed limit on a highway. As the driver approaches an exit, they observe a police car parked on the shoulder with its lights off. Upon seeing the police car, the driver immediately reduces their speed to below the legal limit. In this specific scenario, what does the presence of the police car primarily function as for the driver’s speeding behavior?

A discriminative stimulus SD for punishment is an antecedent stimulus that signals that a particular behavior if emitted will be followed by a punishing consequence In this example the police car serves as an SD for punishment because its presence signals that speeding the behavior is now highly likely to be followed by an aversive consequence such as a traffic ticket or other legal penalties This increased likelihood of punishment is what causes the driver to reduce their speed It is not an Sdelta A because an Sdelta signals that reinforcement or punishment is unavailable While slowing down might lead to avoiding punishment which is a form of negative reinforcement the police cars direct function related to the speeding behavior is to signal the availability of punishment C not primarily to signal reinforcement for slowing down B Furthermore the police car is an antecedent stimulus not a consequence so it cannot be a punisher D

#131. A behavior technician is working on a skill acquisition plan where the client is learning to follow multi-step instructions. The technician provides a vocal antecedent stimulus (SD) such as, ‘Pick up that pencil, then give me a high five.’ The client then physically picks up the pencil and gives a high five, and is reinforced for these accurate, non-verbal responses. What primary type of training is the behavior technician conducting in this scenario?

In this scenario the client is primarily engaged in listener behavior Listener behavior involves responding nonverbally to the verbal behavior of others The client is accurately following multistep vocal instructions Pick up that pencil then give me a high five by performing the requested actions Autoclitic training involves verbal behavior that modifies or elaborates on other verbal behavior Expressive behavior also known as speaker behavior or verbal operants like tacts mands intraverbals echoics refers to the production of verbal responses Intraverbal training specifically involves responding verbally to anothers verbal behavior without pointtopoint correspondence eg answering questions filling in blanks Since the clients responses are physical actions to a verbal SD the training is targeting listener skills

#132. Tony is attempting to reduce his MM consumption at work. A behavior analyst is collecting occurrence data on his interactions with a bowl of MMs. Today, Tony walked by the bowl 20 times. During these occurrences, he took eight MMs, actively avoided looking at the bowl three times, and walked away from the bowl four times without taking any. Based solely on the number of times he walked by the bowl, what percentage of occurrences did Tony not take an MM?

To answer this question it is crucial to focus on the specific data requested and disregard irrelevant information The question asks for the percentage of occurrences Tony did not take an MM specifically based on the number of times he walked by the bowl 1 Identify total occurrences Tony walked by the bowl 20 times This is our denominator for the percentage calculation 2 Identify relevant occurrences taking MMs He took eight MMs This means that out of the 20 times he passed the bowl he engaged in the MMtaking behavior 8 times 3 Calculate occurrences of not taking MMs If he walked by 20 times and took MMs 8 times then the number of times he did not take an MM is the total occurrences minus the occurrences of taking MMs 20 8 12 times 4 Calculate the percentage Divide the number of times he did not take an MM by the total number of times he walked by the bowl then multiply by 100 12 20 100 060 100 60 The information about avoided looking at the bowl three times and walked away four times are distractors for this particular calculation as the question specifically frames the percentage around the act of not taking an MM when walking by the bowl which is directly derived from the total occurrences and the occurrences of taking an MM

#133. According to the principles of Applied Behavior Analysis and the ‘Dead Man’s Test,’ which of the following scenarios describes an event that is not considered a behavior from a behavior analytic perspective?

Behavior analysts define behavior as anything an organism does A crucial criterion for identifying behavior is the Dead Mans Test which states If a dead man can do it it aint behavior This test helps distinguish between actions performed by an organism and events that happen to an organism or internal states A student actively raises their hand is clearly something the student does passing the Dead Mans Test A child is observed lying motionless on the couch fails the Dead Mans Test a dead person can lie motionless This is not considered an active doing by the organism A person is suddenly splashed by a passing car also fails the Dead Mans Test a dead person can be splashed This is an event happening to the person not something the person is doing A client reports feeling intensely hungry refers to an internal state or private event feeling hungry not an observable or measurable doing of the organism in the same way raising a hand is While behavior analysts acknowledge private events like thinking internal physiological processes or subjective feelings like hunger are generally not classified as behaviors themselves for the purpose of direct observation and intervention as the feeling of hunger is something that occurs to the person rather than something they actively do The report of feeling hungry is a behavior a verbal one but the feeling itself is not Given the options the feeling of hunger is the best fit for not considered a behavior based on the texts emphasis on being hungry is not a behavior and internal functions and processes are also not behaviors

#134. A behavior technician is working with a child who frequently throws toys across the room during play time. After an instance of toy-throwing, the technician directs the child to not only pick up the thrown toy but also to organize the entire toy bin, sort all the toys into designated containers, and wipe down the play table. This intervention requires the child to make the environment better than it was before the misbehavior occurred. This intervention best exemplifies which of the following behavior-change procedures?

Restitutional overcorrection is a form of overcorrection where the individual is required to restore the environment to a state better than it was before the inappropriate behavior occurred In this scenario the child not only picks up the thrown toy but also organizes the entire toy bin sorts toys and wipes the table all of which exceed simply returning the environment to its original state Negative practice overcorrection involves engaging in the incorrect behavior repeatedly Positive practice overcorrection involves repeatedly performing a correct form of the behavior Response cost involves removing a specific amount of a reinforcer contingent on the occurrence of a behavior

#135. A behavior analyst is reflecting on their professional responsibilities and ethical obligations to clients, stakeholders, and the field of Applied Behavior Analysis. According to the BACB ethical code, which of the following is a primary obligation of behavior analysts?

The text directly states that behavior analysts have an obligation to act in a way that reflects positively on the field of behavior analysis Yes this is straight from the ethical code straight from the BAC board This obligation ensures that the profession maintains a high standard of conduct and public perception The text also explicitly refutes the idea of guaranteeing outcomes stating We should never guarantee anything Prioritizing dual relationships is contrary to ethical guidelines which mandate avoiding them While behavior analysts should use evidencebased interventions expecting a 100 success rate is unrealistic and not an ethical obligation

#136. Dan’s in-laws are scheduled to arrive in town while he is at work. Upon leaving work, Dan deliberately chooses to take a route home that adds an additional 45 minutes to his usual commute time. Based on this scenario and the principle of parsimony in behavior analysis, what is the most likely function of Dan’s behavior?

The information provided indicates that Dans inlaws are arriving and he takes an unusually long route home delaying his arrival by 45 minutes This action directly leads to a postponement of his interaction with his inlaws suggesting an attempt to escape or avoid this situation According to the principle of parsimony we should choose the simplest and most logical explanation for behavior given the available information There is no evidence in the scenario to support the functions of obtaining a tangible gaining attention or automatic sensory stimulation Therefore the most straightforward hypothesis is that Dan is engaging in escapeavoidance behavior related to his inlaws arrival

#137. Lucy repeatedly tells Charlie that she will hold a football still for him to kick, but Charlie always protests, saying, ‘No, because you always pull the football away before I kick it’ Despite his protests, Lucy usually manages to convince Charlie to attempt the kick anyway. Lucy’s action of pulling the football away just as Charlie attempts to kick it, in the context of its effect on Charlie’s future likelihood of attempting to kick, functions as what?

For an event to be classified as a reinforcer positive or negative or a punisher it must reliably change the future probability of the behavior it follows Reinforcers increase future behavior while punishers decrease it In this scenario despite Charlies past experiences and protests about Lucy pulling the ball away he continues to attempt the kick The text states Lucy is able to get Charlie to do it anyway and he continues to do it This indicates that Lucys action of pulling the ball away has neither decreased the future probability of Charlies kicking attempts ruling out punishment nor increased it beyond what might occur in the absence of this consequence ruling out reinforcement Therefore while it is a consequence that occurs after Charlie attempts to kick the provided information does not demonstrate a functional relationship where it consistently alters the future rate or likelihood of his kicking behavior It is simply a consequence without a demonstrated reinforcing or punishing effect on Charlies future actions

#138. A behavior analyst is designing a new educational program for adult learners who require flexible scheduling and individualized progression through the curriculum. The program aims for high levels of proficiency on each unit before advancing to the next. Which method of instruction is most aligned with these objectives of individual mastery and self-pacing?

Personalized System of Instruction PSI also known as the Keller Plan is a method of instruction characterized by selfpacing mastery learning and a focus on individual performance Learners progress through the curriculum at their own speed demonstrating mastery of one unit before moving to the next The text directly states that Personalized System of Instruction has to do with individual mastery and selfpacing In contrast Incidental Teaching is a naturalistic approach that capitalizes on client interests in the natural environment Precision Teaching focuses on building fluency accuracy and speed using standard celeration charts and Direct Instruction typically involves group instruction choral responding and highly structured lessons

#139. Immediately after his teacher hands him a quiz, Bart becomes agitated and rips the quiz into two pieces. As a consequence, Bart is required to miss recess and, during that time, must rip 100 pieces of blank paper into two pieces each. This intervention requires Bart to engage in the undesired behavior repeatedly. What specific type of punishment intervention is being utilized here?

Negative practice overcorrection is a form of overcorrection where contingent on an inappropriate behavior the individual is required to engage in the undesired behavior repeatedly In this scenario Bart ripped his quiz the undesired behavior and then he was made to repeat the same action ripping paper 100 times Restitutional overcorrection would involve restoring the environment to a state that is better than it was before the transgression eg if Bart had to clean up his ripped paper and also clean other areas of the classroom Positive practice overcorrection involves repeatedly practicing an appropriate or desired alternative behavior eg practicing neatly putting paper away Response cost is a form of punishment where a specific amount of a reinforcer is removed eg losing tokens or points contingent on the occurrence of a behavior

#140. A Board Certified Behavior Analyst (BCBA) is conducting a functional analysis for a client’s self-injurious behavior (SIB). After implementing a function-based intervention that successfully reduced SIB, the BCBA temporarily withdrew the intervention conditions and reintroduced the baseline conditions to observe if the SIB returned to its previous high levels. This phase of the experiment, where the behavior is allowed to revert to pre-intervention levels to confirm the intervention’s effect, is best described as which component of experimental control?

Verification is a crucial component of experimental control particularly within withdrawal or reversal designs eg ABAB designs It involves returning to baseline conditions or removing the independent variable after an intervention has produced a change in behavior to observe if the behavior reverts to its original preintervention level This process serves to confirm that the observed change in the dependent variable the behavior was indeed a function of the independent variable the intervention rather than other extraneous factors The text states verification is when our Baseline data remaining consistent if the IB was never introduced verifying your prediction which implies demonstrating that the behavior returns to its baseline state when the intervention is absent thus verifying the initial prediction of change due to the intervention Prediction is making an educated guess about the outcome Replication involves repeating the experiment to demonstrate reliability Parsimony is an attitude of science not a component of experimental control

#141. A supervising BCBA mentions needing new jeans for spring to a supervisee during a casual chat. Knowing the BCBA’s birthday is approaching, the supervisee surprises them with a pair of jeans from their favorite store. Considering the updated BACB Professional and Ethical Compliance Code, is it permissible for the BCBA to accept this gift, and why?

The BACB Professional and Ethical Compliance Code for Behavior Analysts specifically section 107 Relationships with Clients and Stakeholders and 504 Accepting Gifts clarifies that behavior analysts should not accept gifts from clients or stakeholders with a monetary value of more than 10 and should not accept gifts on a routine or annual basis It is crucial to understand that these same guidelines apply to relationships with supervisees and trainees to prevent potential conflicts of interest dual relationships or the appearance of impropriety within the supervisory relationship While fostering a positive relationship is important ethical boundaries must be maintained A pair of jeans from a favorite store would almost certainly exceed the 10 nominal value limit making it impermissible to accept Supervisors like clients and stakeholders must adhere to these limits to ensure professional boundaries and ethical conduct

#142. A Board Certified Behavior Analyst (BCBA) has a decade of experience working with individuals diagnosed with autism spectrum disorder in home and school settings. This year, under a new contract with a school district, the BCBA is assigned to a classroom that includes students with various disabilities beyond autism, some of which are outside their direct prior experience. Considering the BCBA is contracted through a third-party (the school district), what is the BCBA’s primary ethical responsibility in this situation?

When working under a thirdparty contract a BCBAs primary ethical responsibility remains with the clients receiving services The prompt explicitly states your responsibility is always to the clients your responsibility is to do no harm to those clients and you still need to adhere to the ethical codes A To strictly adhere to the terms and limitations of the contract with the school district above all else While fulfilling contractual obligations is important ethical responsibility to the client always takes precedence especially if theres a potential conflict or if client welfare is compromised The ethical codes of the profession guide how one operates within a contract B To prioritize the welfare and best interests of the clients students and ensure all services provided adhere to the ethical codes of the profession This is the correct answer The BCBA has an ethical obligation to ensure that all services provided are within their scope of competence or that they obtain the necessary training supervision or consultation to become competent with new populations Their ultimate duty is to provide effective ethical treatment always prioritizing the wellbeing and best interests of the students This aligns with the Ethical Principles of Behavior Analysts particularly Responsibility to Clients and Stakeholders and Competence and Professional Development C To politely decline the assignment stating that their experience is limited to autism and request reassignment to a more familiar classroom This might be an option after assessing competence and exploring options for trainingsupervision but its not the primary ethical responsibility The BCBAs initial responsibility is to evaluate if they can competently serve the new population potentially by seeking supervision or further training rather than immediate refusal D To focus solely on the students with autism in the classroom as this is where their competency lies and provide minimal support to other students This approach would be unethical as it neglects the needs of other students assigned to their care potentially denying them necessary services or support If the BCBA is assigned to the entire classroom their ethical duty extends to all students within that assignment either by direct service if competent or by advocating for appropriate servicesreferrals

#143. Prior to a two-week Christmas break, several students in a classroom setting consistently stopped requesting to play with Legos during their free time, despite Legos being a previously highly preferred item. Upon returning from vacation, all of the students immediately began requesting the Legos for break time once again. Considering this sequence of events, what change in motivating operations (MOs) most likely occurred during the vacation period regarding the Legos?

Before the Christmas break the students likely experienced satiation A with Legos meaning prolonged exposure or access to Legos decreased their reinforcing effectiveness and thus reduced the likelihood of requesting them During the twoweek vacation the students were likely deprived D of Legos meaning their access to Legos was restricted or absent Deprivation is an establishing operation EO that increases the current effectiveness of a stimulus as a reinforcer and increases the frequency of all behavior that has been reinforced by that stimulus When they returned from vacation the value of Legos as a reinforcer had increased due to this period of deprivation leading them to request Legos once again Reinforcement B or punishment C cannot be determined without specific information about the consequences of their behavior during the break the question specifically asks what occurred during vacation to change the motivating operation for Legos

#144. Elliot, a doctor at Sacred Heart Hospital, frequently expresses low self-confidence despite her consistent effectiveness as a practitioner. To help her recognize her professional capabilities, her supervisor recommends that Elliot systematically collect and record data on the positive outcomes and successes of her medical procedures. What specific self-management strategy is Elliot’s supervisor recommending?

Selfmonitoring is a selfmanagement strategy that involves a person systematically observing and recording the occurrence of their own behavior In this scenario Elliots supervisor is recommending that she take data on her outcomes to establish a record of her successes which is a direct and precise definition of selfmonitoring The text reinforces this by stating Elliot is taking data shes simply taking the data so what is La supervisor suggesting you do a selfmonitor yes all the supervisor is saying is Monitor yourself right during your procedures during your treatments just take data on outcomes Selfinstruction would involve Elliot verbally guiding herself through tasks Selfevaluation would require Elliot to compare her performance against a preset standard or goal While selfcontrol is a broader term encompassing selfmanagement strategies aimed at changing ones own behavior selfmonitoring is the most specific strategy being employed here as the primary action is data collection without an explicit instruction for immediate behavior change or comparative evaluation

#145. You receive a wedding invitation from your best friend, which includes details about the date, time, and location. Additionally, there is a section where you are asked to select your preferred meal choice, with the only options being ‘fish’ or ‘chicken’. Considering standard behavior analytic preference assessment methods, what type of preference assessment best represents this scenario?

The scenario where you are presented with a choice between fish or chicken directly represents a Paired Stimulus Preference Assessment also commonly known as a Forced Choice assessment In this method two stimuli are presented simultaneously and the individual is asked to choose one This process is repeated with all possible pairs of stimuli to create a hierarchy of preferences The text explicitly links choosing between fish or chicken to a forced choice or paired stimulus type of preference assessment A Single Stimulus or Successive Choice assessment involves presenting one item at a time Multiple Stimulus With Replacement MSW and Multiple Stimulus Without Replacement MSWO assessments involve presenting three or more stimuli simultaneously differing in whether the chosen item is returned to the array MSW or removed MSWO for subsequent trials

#146. For over three years, you have been attempting to place your ex-girlfriend’s calls on extinction, as she calls approximately every two to three months. However, every six months, you inadvertently pick up the phone or text her back. Considering this intermittent and inconsistent application of an extinction procedure, what is the most likely outcome regarding the ex-girlfriend’s calling behavior?

The text explicitly states that if a behavior is intermittently reinforced while attempting extinction we will never reach extinction or extinction is not going to happen Intermittent reinforcement even if infrequent like every six months in this case makes a behavior more persistent and harder to extinguish This phenomenon is known as increased resistance to extinction which refers to the extent to which responding persists after an extinction procedure has been implemented Spontaneous recovery is the reemergence of an extinct behavior but if extinction is not achieved due to intermittent reinforcement spontaneous recovery cannot occur The behavior will not quickly extinguish rather it will be much harder to extinguish Planned ignoring is a form of extinction but it is not successfully implemented if reinforcement is occasionally provided

#147. Jenny needs to gather information on how frequently her son’s daycare takes him to the bathroom throughout each day. She wants to identify the most useful and specific dimensional quantity to accurately track this behavior over time, allowing for comparisons across different days or weeks. Considering the core dimensional quantities of behavior, which measure would be most appropriate for Jenny’s objective?

Jennys objective is to determine how often her son goes to the bathroom each day which inherently includes a temporal component While frequency a simple count of occurrences is a measure of repeatability rate is a more precise and often more useful dimensional quantity when a specific time component is involved Rate is defined as the number of responses or occurrences of a behavior per unit of time eg responses per minute per hour or per day By expressing the bathroom trips as times per day Jenny obtains a rate This measure is superior to simple frequency in this context because it standardizes the count by the observation period allowing for meaningful comparisons eg he went 6 times per day on Monday but 4 times per day on Tuesday Without the time component frequency alone eg six times would lack context Duration measures the total time an event occurs and latency measures the time between a stimulus and the onset of a response neither of these directly addresses how often per day within a specified period

#148. Kate, a BCBA, is reviewing frequency data for a client’s hand-flapping behavior. She notices a significant increase in the recorded frequency over the past week, but the RBT, John, has not reported any changes in the client’s behavior or environment. During a supervision session, Kate observes John recording not only hand-flapping but also instances of rapid arm movements that are distinct from the operational definition of hand-flapping. What is the most likely explanation for the discrepancy in the data?

Observer drift refers to an unintended change over time in the way an observer applies an operational definition of a behavior This can occur when observers gradually alter their interpretation of the definition become less stringent or start including behaviors that were previously excluded or viceversa In this scenario Kate observes John recording not only handflapping the target behavior but also rapid arm movements that are distinct from the original operational definition This indicates that Johns understanding or application of the definition has changed over the week leading to an inflation of the frequency data and thus a data discrepancy Reactivity A Reactivity occurs when the presence of an observer influences the behavior of the observed individual While Kates presence could influence John the data discrepancy started a week ago before Kates direct observation during supervision The issue is a change in measurement criteria over time not a direct behavioral response to being observed Treatment drift C Treatment drift occurs when the independent variable the intervention or treatment procedure is implemented inconsistently or incorrectly over time The question focuses on measurement discrepancies and the application of an operational definition not changes in the treatment protocol itself Naive observer D A naive observer is someone who is unaware of the purpose of an experiment or the expected outcomes The RBT John is a trained professional and is aware he is collecting data The issue isnt a lack of knowledge or awareness but a subtle unintended alteration in the application of his knowledge over time Therefore observer drift is the most accurate explanation for the described data discrepancy Regular supervision and booster training are crucial for preventing and correcting observer drift

#149. An RBT is teaching a client about different types of communication. Which of the following scenarios best exemplifies point-to-point correspondence, as described in the provided text?

Pointtopoint correspondence means that the beginning middle and end of the verbal stimulus match the beginning middle and end of the verbal response This implies that the form of the stimulus and the form of the response are identical In option B the clients verbal response Jump up and down exactly matches the RBTs verbal stimulus Jump up and down This is a clear example of an echoic which by definition involves pointtopoint correspondence and formal similarity Option A is a tact seeing a sign and stating its meaning Option C is a tact identifying an objects color Option D is also a tact labeling an object While tacts can have formal similarity with the listeners response they do not have pointtopoint correspondence with the nonverbal stimulus that evokes them The text uses the example of reading No Trespassing and saying No trespassing to illustrate this exact match which is what option B demonstrates with spoken words

#150. A Board Certified Behavior Analyst (BCBA) is designing an intervention for a young learner with developmental delays to teach independent handwashing. The BCBA aims to prevent errors from occurring during the acquisition phase of this new skill. Which of the following scenarios best exemplifies the application of errorless learning?

Errorless learning is a teaching strategy designed to prevent a learner from making mistakes during the acquisition of new skills The core principle is proactive intervention to guide the learner to the correct response thereby preventing errors from occurring and becoming part of a behavioral chain In the context of a BCBA teaching handwashing providing full physical handoverhand guidance throughout the entire sequence Option C is a direct application of errorless learning This method ensures that the learners responses are always correct because the prompt is so intrusive that it essentially does the behavior for the learner allowing for immediate reinforcement of a correct response Lets evaluate the other options Option A Backward chaining with errors in earlier steps While backward chaining is a valid procedure if reinforcement is only at the end and errors are allowed in earlier steps it does not fit the definition of errorless learning Errorless learning would require preventing those errors in the initial steps Option B Waiting for an attempt then prompting This scenario describes a leasttomost prompting strategy or an errorcorrection procedure where errors are allowed to occur before a prompt is introduced This is not errorless learning as errors are permitted Option D Waiting for initiation then verbal prompt Similar to Option B this approach allows for a period where the learner might make an error or no response before a prompt is given Errorless learning aims to prevent even these momentary opportunities for error

#151. A newly certified behavior analyst designs a highly effective intervention plan for several individuals residing in a group home, leading to significant positive changes in their target behaviors. However, the plan is written using numerous specialized acronyms and abbreviations that are not commonly understood by other behavior analysts and support staff at the facility. If this behavior analyst were to leave or transfer, other professionals would struggle to accurately implement the intervention. Which dimension of Applied Behavior Analysis (ABA) has been violated by the behavior analyst’s treatment plan documentation?

The seven dimensions of ABA as outlined by Baer Wolf Risley 1968 serve as foundational guidelines for practice In this scenario the dimension of Technological has been violated A technological intervention means that the procedures are described clearly and concisely in sufficient detail so that other trained individuals can accurately replicate the intervention The use of confusing acronyms and abbreviations hinders replication and consistent implementation by other professionals even if the intervention itself is effective Lets review the other options Applied Focuses on the social significance of the target behaviors and the interventions relevance to everyday life The question implies the intervention targets socially significant behaviors so this dimension is likely met Effective Requires that the intervention produces significant practical and meaningful changes in behavior The question explicitly states the plan is highly effective so this dimension is met Conceptually Systematic Means that the intervention is derived from and consistent with the basic principles of behavior analysis While not explicitly stated the question does not suggest a lack of conceptual grounding but rather a problem with its description

#152. Greg’s mother successfully taught him how to complete the entire sequence of mopping the floor using a forward chaining strategy. During the initial teaching phase, she always ensured the mop was readily available and the mop bucket was already filled with water and cleaning solution before Greg started. Now that Greg has mastered the mopping skill and performs it independently, his mother intentionally leaves the mop bucket empty, requiring Greg to fill it himself before he can begin mopping. She observes to see if Greg will attempt to problem-solve (e.g., look for the water source, find soap) or ask for help to complete the task. What strategy is Greg’s mother employing by not filling the mop bucket?

Gregs mother is employing a Behavior Chain Interruption Strategy BCIS This strategy is specifically utilized after a behavior chain has been completely mastered and is performed fluently by the individual It involves deliberately interrupting the established chain at a specific point by altering or omitting an expected antecedent or material for a subsequent step The purpose of BCIS is not to teach the chain itself but to evoke novel behaviors such as problemsolving manding requesting assistance or items or other adaptive responses when the familiar sequence is disrupted This promotes flexibility generalization of skills and independent problemsolving rather than rote performance Forward chaining backward chaining and total task chaining are all methods for teaching a new behavior chain by sequencing steps and providing reinforcement which is distinct from the described intervention that occurs after mastery to promote adaptability

#153. Jack has spent the entire morning in classes and is feeling extremely hungry. As he and his friends drive around looking for a place to eat, Jack spots a sign for a Chinese restaurant. Immediately upon seeing the sign, he exclaims, ‘Chinese’ Given that Jack is experiencing significant food deprivation (MO) and is responding to the visual stimulus of the restaurant sign (non-verbal stimulus), what type of verbal operant is Jack’s utterance ‘Chinese’?

Jacks utterance Chinese in this context is an example of an impure tact An impure tact is a verbal operant that is evoked by a motivating operation MO AND a nonverbal discriminative stimulus SD and it has pointtopoint correspondence with the stimulus In this scenario Jacks hunger food deprivation serves as the motivating operation MO and the visual stimulus of the Chinese restaurant sign is the nonverbal SD His verbal response Chinese is controlled by both of these variables It is not a mand because he is not requesting Chinese food he is labeling what he sees It is not an intraverbal because it is evoked by a nonverbal stimulus not a verbal SD and it has pointtopoint correspondence with the stimulus It is not transcription because he is not writing or typing a verbal stimulus

#154. Selena, a Board Certified Behavior Analyst (BCBA) managing a caseload of five clients, decides to take a six-month sabbatical to Spain. She understands that discontinuing services or taking extended leave requires careful ethical consideration to ensure client well-being. What is the most appropriate and ethically responsible action Selena should take before her departure for a six-month sabbatical?

The text directly addresses this scenario stating Is it fair to the clients that their services are paused because Selena is leaving for six months No Thats not going to be the best most ethical way to handle this Okay You cant just leave This clearly rules out simply pausing services Option A or expecting clients to wait Option B The ethical code emphasizes continuity of care When a behavior analyst must take an extended leave they have an obligation to facilitate the smooth transfer of clients to other competent professionals or to ensure that clients needs are met through other appropriate arrangements Option C best reflects this ethical responsibility prioritizing the clients ongoing access to necessary services and minimizing disruption to their treatment

#155. A behavior analyst is designing a high-probability (High-P) request sequence to increase compliance with a low-probability (Low-P) request, ‘Clean your room.’ Which of the following sequences best exemplifies the correct application of a High-P Low-P sequence, considering the principles discussed in ABA?

Option B is the most appropriate HighP LowP sequence The text emphasizes introducing 25 highprobability requests touch your nose clap your hands stomp your feet that are easy and rapidly complied with immediately followed by the lowprobability request clean your room Options A introduces spell your name which is likely a more difficult potentially lowprobability request inserted before the target lowprobability request violating the principle of maintaining momentum with highP tasks Options C and D include pauses or waits between requests which directly contradicts the goal of building and maintaining behavioral momentum rapidly as pauses can break the momentum and reduce the effectiveness of the intervention The core idea is to keep the momentum going without interruption

#156. The text briefly differentiates various behavioral processes, noting that ‘responding conditioning is stimulus response has to do with reflexes.’ Based on this description, which of the following scenarios is the most fitting example of a behavior primarily established or explained by respondent conditioning?

The text defines respondent conditioning as involving stimulus response has to do with reflexes This highlights that respondent conditioning deals with involuntary reflexive behaviors that are elicited by specific stimuli either unconditioned innate or conditioned learned through association The scenario where a person experiences an increased heart rate and sweaty palms when encountering a snake is an example of an autonomic reflexive response fear that is typically an unconditioned response to a fearinducing stimulus or a conditioned response if the individual has learned to associate snakes with danger Option A A child learns to say please to request a desired toy describes an operant behavior Saying please is a vocal response that is strengthened by the consequence of receiving the toy which is a form of positive reinforcement Option B A student diligently studies for an exam because previous good grades resulted from similar study habits also describes operant behavior Studying is a behavior maintained by the reinforcing consequence of good grades Option D An individual avoids touching a hot stove after experiencing pain from a previous contact with it is an example of operant behavior maintained by negative punishment or direct avoidance learning The pain from touching the stove serves as a punisher decreasing the likelihood of future touching or the avoidance is reinforced by the absence of pain While there might be a reflexive withdrawal during the initial contact the avoidance of touching it later is an operant contingency

#157. A BCBA is conducting a preference assessment for a new client and needs to determine a clear hierarchy of preferred items, ranking them from most to least preferred. Which type of preference assessment is generally considered the most effective for establishing such a detailed hierarchy by comparing each item against every other item?

The goal is to determine a hierarchy of preference meaning a ranked order of items from most to least preferred based on systematic comparison Forced Choice Paired Stimuli Assessment involves presenting two items at a time and asking the individual to choose one This systematic comparison of every item to every other item in the array provides the most comprehensive data for establishing a clear preference hierarchy The number of times each item is chosen over others can then be used to rank them accurately This method is highly effective because it directly assesses relative preference between all possible pairs Multiple Stimulus With Replacement MSW involves presenting an array of items allowing the individual to choose one returning the chosen item to the array and then rearranging the remaining items and the chosen item for the next trial While it provides preference data it does not systematically compare each item to all others in the same way Forced Choice does for a precise hierarchy Multiple Stimulus Without Replacement MSWO involves presenting an array of items allowing the individual to choose one and then removing the chosen item from the array for subsequent trials This results in a ranked order of selection but it doesnt involve the direct repeated comparison of every pair of items that Forced Choice does The choices made earlier in an MSWO can influence subsequent choices from a diminishing set of options Single Stimulus Successive Choice involves presenting one item at a time and measuring the individuals approach or engagement with it It identifies items that are preferred but does not establish a hierarchy relative to other items only whether a single item is preferred or not in isolation

#158. A behavior analyst has been observing a client’s disruptive behavior for several weeks. Initially, they defined disruptive behavior as any vocalization louder than a whisper or any instance of leaving their seat without permission. Over time, the analyst’s interpretation of disruptive behavior subtly changed, and they began to consistently record instances of fidgeting as disruptive, even though it wasn’t part of the original operational definition. This gradual and unacknowledged alteration in the measurement criteria is most accurately described as

Observer drift occurs when observers unknowingly alter the way they apply a measurement system leading to a gradual and subtle shift in how they interpret or record target behaviors over time This can cause their data to deviate from the true occurrence of the behavior compromising the validity and reliability of the data In this scenario the analysts definition of disruptive behavior changed from the initial operational definition to include fidgeting which was not originally part of the criteria Observer bias refers to a tendency for observers to see what they expect to see Reactivity refers to a change in behavior due to being observed IOA inconsistency is a result of rather than the cause of the changing criteria

#159. A Board Certified Behavior Analyst (BCBA) is implementing a changing criterion design to reduce the number of cigarettes smoked by a client. The initial criterion is set at 20 cigarettes per day, and the BCBA plans to systematically decrease this criterion over time. To confidently demonstrate experimental control within this design, which of the following outcomes is most crucial?

In a changing criterion design experimental control is demonstrated when the target behavior systematically changes in a stepwise fashion to match the new criterion set for each phase This means if the criterion is decreased the behavior should decrease to meet it if the criterion is increased the behavior should increase to meet it Option C accurately reflects this by stating the behavior changes concurrently ie in the same direction and magnitude or at least to the criterion level with the criterion Options A and B are incorrect because drastically below the criterion or simply changing without matching the criterions direction or level does not demonstrate control Option D indicates a lack of control as the behavior is not conforming to the interventions goals

#160. A client has successfully learned to write their name independently, but this skill is currently restricted to using only a Sharpie. They are unable to perform the same task with a crayon, pen, or pencil. To address this, the behavior analyst introduces various writing utensils during subsequent sessions, requiring the client to write their name with each. What behavioral process is the behavior analyst primarily attempting to train or promote?

Stimulus generalization occurs when a learned response occurs in the presence of stimuli that are different from the original training stimulus but share some common properties In this scenario the client has learned one response writing their name but it is currently under the control of only one specific stimulus a Sharpie The goal of the intervention is to ensure that the same response writing their name occurs across multiple different stimuli crayon pen pencil This directly aligns with the definition of stimulus generalization Response generalization on the other hand involves one stimulus evoking multiple varied responses Stimulus equivalence refers to the emergence of accurate responding to untrained and unreinforced stimulusstimulus relations following the reinforcement of some stimulusstimulus relations eg reflexivity symmetry transitivity Compliance while a desired behavioral outcome is not the specific behavioral process of generalization being targeted

#161. A behavior analyst is supervising a new RBT who is collecting data on a client’s hand-flapping behavior using partial interval recording with one-minute intervals. After reviewing the RBT’s data, the behavior analyst observes that the data indicates hand-flapping occurred during 7 out of 10 intervals in a 10-minute session. Based on the principles of discontinuous measurement, specifically partial interval recording, what common measurement distortion is MOST likely to be represented by this data?

Partial interval recording PIR is a discontinuous measurement procedure where an observer records if a behavior occurs at any point during a specified interval Due to its nature if the behavior occurs even for a single moment within the interval it is marked as having occurred for the entire interval This characteristic leads to an overestimation of the behaviors total duration because a brief occurrence is counted as an occurrence for the full interval Conversely PIR often underestimates the total frequency or rate of behavior If a behavior occurs multiple times within a single interval it is only recorded once for that interval meaning the actual count of occurrences is missed For example if handflapping occurs for 5 seconds at the start of a 1minute interval and then again for 30 seconds later in the same interval PIR would only mark it as occurred once thus underestimating the frequency and overestimating the duration as it attributes a 5second or 30second behavior to the full minute

#162. Kendall is participating in a matching-to-sample procedure. First, he is presented with a picture of a dog as the sample stimulus and successfully matches it to an identical picture of a dog. Next, he is given a picture of a cat as the sample and matches it to an identical picture of a cat. Finally, he is given a picture of a tiger and matches it to an identical picture of a tiger. Based on these observations, what property of stimulus equivalence has Kendall demonstrated?

Kendall has demonstrated reflexivity Reflexivity also known as identity matching or generalized identity matching occurs when an individual matches a stimulus to itself AA without prior direct training It is the ability to select a comparison stimulus that is the same as the sample stimulus In the given scenario A picture of a dog Stimulus A is matched to an identical picture of a dog Stimulus A a picture of a cat Stimulus B is matched to an identical picture of a cat Stimulus B and a picture of a tiger Stimulus C is matched to an identical picture of a tiger Stimulus C This consistent matching of identical stimuli directly exemplifies the AA relationship which defines reflexivity Symmetry involves demonstrating that if AB then BA eg if a spoken word dog A matches a picture of a dog B then the picture of a dog B also matches the spoken word dog A Transitivity involves demonstrating a derived untrained relationship between two stimuli AC after having trained two other conditional relations AB and BC For example if a spoken word dog A matches a picture of a dog B and the picture of a dog B matches the written word DOG C then the spoken word dog A will also match the written word DOG C Reactivity refers to the phenomenon where an individuals behavior changes when they know they are being observed or assessed which is unrelated to the properties of stimulus equivalence

#163. After observing a track star achieve an exceptionally fast time in a race, an opposing coach expresses significant skepticism, stating, ‘I don’t believe that time I want to see that happen again.’ The other coach defends the result, pointing out that it is objective, observed data. Which philosophical assumption of science is the opposing coach primarily exercising by questioning the findings and demanding a re-demonstration or replication, even when presented with empirical data?

Philosophical doubt is a core attitude of science that requires scientists to continually question the truthfulness of scientific findings and to remain open to new discoveries and skepticism even when data seems clear It emphasizes the importance of replication and thorough scrutiny of results Empiricism refers to objective observation and measurement of phenomena Determinism is the assumption that the universe is a lawful and orderly place where all phenomena occur as a result of other events Selectionism describes the processes by which behavior is shaped by its consequences ontogeny and by natural selection phylogeny The coachs skepticism and demand for replication despite the presented data directly exemplify philosophical doubt

#164. A Board Certified Behavior Analyst (BCBA) is working with a learner on identifying musical instruments. Initially, the BCBA teaches the learner to say guitar when presented with a picture of an acoustic guitar. After several successful trials, the learner begins to spontaneously label the same picture of the acoustic guitar as string instrument and folk instrument, even though these specific labels were never directly taught for that particular picture. Which behavioral phenomenon is being demonstrated by the learner’s expanded labeling?

Response generalization occurs when a learner emits varied but functionally equivalent responses to a single stimulus or stimulus class even though only one response was directly taught In this scenario the single stimulus picture of an acoustic guitar evokes multiple related responses string instrument folk instrument beyond the initially taught response guitar This phenomenon is characterized by multiple responses occurring to a single stimulus This contrasts with stimulus generalization where a single response is emitted across a range of different but similar stimuli Maintenance refers to the continued performance of a learned behavior after intervention has ceased which is not the primary focus here Response differentiation is the process of reinforcing only those responses that meet a specific criterion leading to a more precise or specific response which is the opposite of what is observed here as the responses are expanding not narrowing The text provides a clear analogy with the yoyo trick teaching walk the dog and then the learner performing around the world and cats cradle with the same yoyo demonstrating multiple responses to a single stimulus which is the definition of response generalization

#165. A client has a strong desire for a specific type of cookie, having not eaten for several hours. However, they only ask for the cookie when they see the cookie jar placed on the counter. In this scenario, what distinguishes the client’s strong desire for the cookie from the cookie jar’s presence?

A motivating operation MO alters the effectiveness of a reinforcer and alters the frequency of behavior that has been reinforced by that reinforcer In this case not eating for several hours an establishing operation a type of MO makes the cookie more valuable as a reinforcer and increases the likelihood of behaviors like asking that have previously resulted in getting cookies A discriminative stimulus SD on the other hand signals the availability of reinforcement for a particular behavior The cookie jar on the counter indicates that cookies are currently available and therefore asking for them is likely to be reinforced The text clearly states You can be motivated to have something but unless theres an SD its not available Thats the key difference between MOs and SDs

#166. Glenn, a BCBA, has identified Hershey Kisses as a highly motivating edible reinforcer for his client during initial assessments. However, when Glenn suggests using chocolate, the client’s parents express significant concern about the potential health risks, specifically cavities, and explicitly state they do not want sugary treats used as reinforcers. Considering ethical guidelines for client-centered practice and the crucial input of stakeholders, how should Glenn most appropriately and professionally proceed in this situation?

The text emphasizes that BCBAs should not go against the parents wishes and that parents are involved in the treatment planning process Therefore directly using chocolate despite protests or circumventing their wishes options A and B is unethical and unprofessional While identifying alternative reinforcers is always a good practice the specific strategy highlighted in the text as a good strategy for this scenario is to request that chocolate be used temporarily each time chocolate is delivered also deliver praise and a token with the explicit aim of attempting to create some backup secondary reinforcers right some conditioned reinforcers through pairing This approach respects parental concerns by making the use temporary and strategic while also building a foundation for more sustainable nonedible reinforcers aligning with best practices for reinforcer fading and ethical stakeholder collaboration Option D is manipulative and disregards parental rights

#167. As a child, your parents warned you that eating too much candy would make your teeth fall out. Even though your teeth never actually fell out from eating candy, you, as an adult, still consistently avoid eating excessive amounts of candy. This avoidance behavior persists without a history of direct punishment from the consequence (teeth falling out). This type of behavior control is best classified as an example of what?

The scenario describes a situation where a verbal statement eating too much candy would make your teeth fall out controls behavior even though the stated consequence your teeth never fell out never actually occurred The text explicitly states if we have a verbal statement that controls our behavior what do we call that do we call that a rule well yes right this is a verbal statement that is controlling our behavior its not contingencybased because theres no consequence right theres never a consequence of your teeth falling out it was enough just to hear that verbal statement that it could happen and thats controlling your behavior making it a rule Option A contingencyshaped behavior is incorrect because this type of behavior is established and maintained by direct contact with consequences in the environment In this example there was no direct consequence for eating candy that led to the avoidance thus it is not contingencyshaped Option B respondent conditioning is incorrect Respondent conditioning involves involuntary reflexlike responses elicited by antecedent stimuli as the text notes responding conditioning is stimulus response has to do with reflexes Avoiding candy is an operant behavior not a reflexive one Option D promptdependent behavior is incorrect While the initial warning acted as a verbal prompt the behavior has persisted and generalized across time and settings indicating it is not dependent on the immediate presence of a prompt but rather governed by the internalized rule

#168. A behavior technician is collecting data on a client’s independent requests for a preferred item. When reviewing a graph of this data, the BCBA observes the data points from day 5 through day 8 and notes a clear downward slope. How would the BCBA most accurately describe the trend of this data for days 5 through 8?

Trend refers to the overall direction of the data path An accelerating trend indicates an upward slope increasing a decelerating trend indicates a downward slope decreasing and no trend or flat indicates that the data path is relatively level Given that the BCBA observes a clear downward slope for days 5 through 8 the most accurate description of the trend is decelerating or decreasing Accelerating and increasing refer to an upward trend while no trend would imply a relatively flat or stable data path

#169. A behavior analyst is reviewing different forms of documentation and data collection. The concept of ‘transcription’ is specifically discussed as the process of writing down verbal stimuli. Based on this understanding, which of the following actions would NOT be considered an example of transcription?

Transcription within the context of verbal behavior and data recording refers to writing down spoken words or more broadly converting a verbal stimulus typically auditory into a written form The defining characteristic is that the source stimulus is verbal and the response is written with pointtopoint correspondence between the two A court reporter typing what is said verbal stimulus to written response is a classic example of transcription A student taking notes on a lecture verbal stimulus to written response is also transcription Writing down a phone number or address spoken to you verbal stimulus to written response is transcription However a chef copying a recipe from a cookbook involves reading a nonverbal visual textual stimulus and then writing This process is more akin to copying or textual responding where both the stimulus and response modes are textual or a form of textual behavior reading followed by writing rather than transcription which specifically involves a verbal spoken stimulus being converted to a written form The provided text explicitly clarifies this distinction by stating typing out a recipe that you read on the internet no you read on the internet okay that nonverbal sd were not transcribing it

#170. A Board Certified Behavior Analyst (BCBA) is overseeing a team of Registered Behavior Technicians (RBTs) who are implementing a new discrete trial training program. To ensure the RBTs are delivering the intervention as intended, the BCBA decides to assess treatment fidelity. Which of the following actions represents a direct form of measurement for assessing the RBTs’ treatment fidelity?

Direct measurement involves observing and recording behavior as it occurs providing immediate and objective data In this scenario attending and directly observing RBTled sessions allows the BCBA to see firsthand whether the treatment plan steps are being implemented correctly which is a direct assessment of treatment fidelity Reviewing data sheets interviewing stakeholders and using selfassessment checklists are all forms of indirect measurement as they rely on secondhand information a product of behavior or selfreport rather than direct realtime observation of the behavior in question the RBTs implementation

#171. A BCBA is training a group of new employees on how to accurately complete session notes for their company. These employees have prior experience working as RBTs in other companies and are generally quick learners. The BCBA observes that they already know most of the steps involved in writing session notes but need to learn the specific nuances and format required by the new company. Considering the employees’ existing skills and learning pace, which chaining procedure would be the most efficient and appropriate for teaching them the company’s specific note-writing protocol?

Total Task Chaining is the most appropriate and efficient chaining procedure when the learner already possesses most of the component skills or can learn quickly In this scenario the employees have prior experience and are quick learners meaning they likely know the majority of the steps for writing notes Total Task Chaining involves teaching the entire chain of behaviors at once allowing the instructor to provide support and reinforce each step as needed This approach is more efficient than forward or backward chaining when the learner has prerequisite skills as it avoids unnecessarily teaching steps the learner already knows Forward and backward chaining are typically used when the learner needs to master steps sequentially often starting from the beginning forward or the end backward of a complex skill which is less necessary here Shaping involves reinforcing successive approximations of a target behavior which is not applicable as the employees are not learning a new motor skill from scratch but rather adapting an existing skill to a new context

#172. A BCBA develops a behavior intervention plan for a client and instructs three RBTs to collect data on the client’s ‘tantrum behavior’ and then calculate the average frequency or duration. RBT 1 submits data indicating that ‘tantrums occurred on average 30 minutes apart.’ RBT 2 submits data stating ‘tantrum duration averaged 3 minutes per occurrence.’ RBT 3 submits data indicating ‘running away happened for 30 seconds’ and reports it as ‘tantrum data.’ This collection of data, when considered across all three RBTs, most clearly lacks what essential characteristic of quality data in Applied Behavior Analysis?

Validity refers to the extent to which a measurement system actually measures what it purports to measure The BCBA asked for data on tantrum behavior RBT 1 reports data on the interresponse time IRT between tantrums occurred on average 30 minutes apart While IRT can be a measure its not explicitly tantrum behavior itself RBT 2 reports data on tantrum duration which directly measures the property of the tantrum RBT 3 however reports data on running away and labels it as tantrum data This indicates a fundamental lack of clarity or agreement on the operational definition of tantrum behavior leading to the measurement of a different behavior entirely When data collectors are measuring different aspects of a behavior or even entirely different behaviors the data lacks validity because it is not consistently measuring the intended target behavior Accuracy refers to the extent to which the observed value matches the true value of the event While there might be accuracy issues within each RBTs data the primary issue highlighted by the different types of data being collected for the same target behavior and one RBT measuring a different behavior altogether is a lack of validitythe collective data isnt measuring tantrum behavior consistently or correctly across all RBTs Reliability refers to the extent to which a measurement procedure yields the same results when repeated under the same conditions eg by different observers or at different times While the data would likely also be unreliable due to the validity issues the root cause here is that they are not even consistently measuring the same thing If all RBTs were measuring tantrum duration but reporting wildly different durations for the same occurrences that would be a primary reliability issue Precision refers to the exactness or refinement of a measurement While important the fundamental issue here is not the level of detail or exactness of the measurement but rather whether what is being measured is truly the target behavior as defined by the BCBA

#173. The Florida State Seminoles football team implements a comprehensive motivational system. Players earn coveted helmet stickers for significant achievements, such as making big plays on the field or achieving high academic marks. However, if players incur penalties for misbehavior during games, fail to meet team duties, or violate team rules, they are required to forfeit or lose previously earned helmet stickers. The specific act of players losing these earned helmet stickers as a direct consequence of their misbehavior is an example of which behavioral principle?

The question specifically asks about the act of losing helmet stickers for getting in trouble or fail to fulfill their duties This describes negative punishment Negative punishment involves the removal lose or forfeit of a stimulus the helmet stickers which are presumably valued and serve as a reinforcer from the environment contingent upon a behavior with the intended effect of decreasing the future probability of that behavior The first sentence describes positive reinforcement adding stickers for good behavior but the question focuses on the second scenario Negative reinforcement involves the removal of an aversive stimulus to increase a desired behavior which is not whats happening here Extinction involves withholding reinforcement for a previously reinforced behavior leading to its decrease but in this case a stimulus is actively being removed not just withheld

#174. A BCBA sets up a treatment room with a table and two chairs. On the table, three pictures are placed a duck, a bear, and a fish. The BCBA hands the client a picture of a duck and says, ‘Put it where it goes.’ The client takes the picture of the duck and places it on the matching picture of the duck already on the table. This demonstration best illustrates which of the following stimulus equivalence relations?

This scenario directly illustrates reflexivity Reflexivity also known as identity matching is a type of stimulus equivalence where a learner matches a stimulus to itself AA without any specific training In this case the client is given a picture of a duck A and matches it to an identical picture of a duck A on the table Symmetry AB so BA would involve a reversible relationship such as if the client was taught to match a picture of a duck to the spoken word duck and then without direct training could match the spoken word duck back to the picture Transitivity AB BC so AC involves untaught stimulusstimulus relations emerging from two other taught relations for example if AB and BC are taught then AC emerges without direct training Expressive identification is a receptive language skill where the learner points to or selects an item when given its name eg Point to the duck but it is not a direct measure of stimulus equivalence or a relation between stimuli in the same way as reflexivity symmetry or transitivity

#175. During an exam, Timmy is sitting quietly at his desk. When the teacher says, ‘Okay, begin your exam,’ it takes Timmy 20 seconds to start answering the first question. He then proceeds to answer questions at an average rate of two minutes per question, completing the entire exam in 30 minutes. Based on the provided text, which specific duration in this scenario best represents the concept of latency?

The text defines latency as the time between the SD and the start of a response In this example the discriminative stimulus SD is the teachers instruction Okay begin your exam The start of a response is when Timmy begins answering the first question The 20second interval between these two events precisely fits the definition of latency Option A describes a period before the SD Option C describes the duration of a response or an interresponse time IRT if it refers to the time between the end of one question and the start of the next Option D represents the total duration of the entire behavior or activity

Previous
Finish
💡 Tip: Like this site?
Bookmark this site using Ctrl + D or tap ‘Add to Favorites’ on your mobile browser.

Popular Categories



Search the website